SlideShare a Scribd company logo
1 of 52
TEST 4
Cám ơn bạn: "Nam Huy" <bui.huy.nam.1990@gmail.com> đã đánh máy bài test này.
LEVEL: PRE-INTERMEDIATE & INTERMEDIATE
I. Choose the word or phrase which best completes each sentence.
1. My mother can’t ____ seeing me at home all day.
a. stop c. give up
b. stand d. suffer
2. When the company had to lay off a number of workers, he became _____
a. considerate c. delyberate
b. redundant d. dismissed
3. Dr. Johnson is a very _______ man in our neighborhood.
a. impossible c. unpopular
b. unlike d. disliked
4. This time next week they ________ to the United States.
a. will be flying c. are flying
b. will fly d. have flown
5. My uncle Tom,_______ you met the other day, is one of the best lawyers in the city.
a. whose c. whom
b. that d. both b and c are correct
6. Mike always keeps his promise, so you can rely ___ him.
a. to c. on
b. for d. at
7. They asked me ______ I had read “Moby Dick” by Herman Melville.
a. what c. whether
b. if d. both a and c are correct
8. I will have your bicycle ______ before returning it to you.
a. repaired c. repairing
b. to repair d. repair
9. Only when you are old enough ________ the truth.
a. do you know c. will you know
b. you do know d. you know
10. We are ______ listening to her advice.
a. interested with c. tired in
b. bored in d. tired of
11. He has been a war ______ for many years and covered the war in Rwanda last year.
a. correspondent c. editor
b. newsman d. reporter
12. She was ________ in the countryside and sent to a little village school.
a. brought on c. taught
b. brought up d. learned
13. The meeting was attended by nearly one hundred scientists, several of ____ were very young.
a. them c. who
b. those d. whom
14. He doesn’t write to his parents as often as he ______
a. used to c. got used to
b. was to d. had to
15. My hair needs _____ , but I’ve been too busy to have it cut.
a. being cut c. cutting
b. to cut d. cut
16. Had they arrived at the fair early, they ________ what they wanted.
a. would have found c. had found
b. found d. find
17. After having been held ____ for 5 years, he was released yesterday.
a. passive c. pensive
b. active d. captive
18. You cannot ______ the truth when that detective questions you yesterday.
a. release c. contain
b. reveal d. captive
19. There is great ____ in climbing Mr.Everest, but many people have made the attempt.
a. trial c. drill
b. pause d. peril
20. His parents are trying to ______ him to get married.
a. cause c. deserve
b. determine d. persuade
II. Read the passage and then choose one answer for each of the following questions:
Chess must be one of the oldest games in the world. An Arab traveller in India in the year 900 wrote that it was
played “long, long ago”. Chess was probably invented in India, and it has been played everywhere from Japan
to Europe since 1400. The name “chess” is interesting. When one player is attacking the other’s King, he says
in English, “Check”; when the King has been caught and cannot move anywhere he says “Check mate”. These
words come from Persian. “Shah mat” means” the king is dead”, that is when the game is over and one player
has won.
Such an old game changes very slowly. The ruler have not always been the same as they are now. For example,
at one time the queen could only move one square at a time. Now she is the strongest piece on the board. It
would be interesting to know why this has happened! Chess takes time and thought, but it is a game for all
kinds of people. You don’t have to be a champion in order to enjoy it. It is not always played by two people
sitting at the same table. The first time the Americans beat the Russians was in a match played by radio. Some
of the chess masters are able to play many people at the same time. The record was when one man played 400
games! It is said that some people play chess by post. This must make chess the slowest game in the world.
1. Which of the following is known to be true?
a. Chess is an old Indian travelling game
b. Chess is the oldest game in the world
c. Chess was played in Japan and Europe before 1400
d. Chess was played in India long before 900
2. One player has won the game when the other player’s King can not move anywhere.
a. he attacks the other player’s King
b. he says some Persian words
c. the other player’s King can not move anywhere
d. he says “check” to the r other player
3. According to the old rules of the game the queen could move no more than one square at a time.
a. the queen was the attacked all the time
b. the King had to attacked all the time
c. the queen could move no more than one square at a time
d. the king could not move anywhere
4. Which of the following will you hear when one player has won the game?
a. “Shah mat” c. “the King is dead”
b. “check” d. “check mate”
5. Which of the following is NOT correct?
a. All kinds of people can play chess.
b. only two people can play chess sitting at the same table.
c. Some people write each other playing chess.
d. The Russians lost the game player by radio.
III. Read the article below and put a cross on the letter next to the word that best fits each space.
Should smoking be banned in public laces?
Statistics (1) show beyond doubt that cigarette smoking can (2) damage the health, yet a surprisingly large
number of people continue to smoke (3) in spite of all warnings. By doing so they are not just shortening their
own lives, they are also affecting the health of (4) those around them. It is time that non-smokers fought bach!
Personally, I think smoking should (5) definitely be banned in public places. In the first place, it is very
unpleasant (6) to sit in a smoke-filled room, such as a restaurant or cinema, if you do not yourself smoke (7)
added to this, smoking can be a serious fire risk, especially in crowed places (8) like discos. Finally, in my
opinion, nobody should be asked to risk his heath just because of another person’s bad habits.
Smokers may (9) protest that they should be free to do as they like. They say that we already have no-smoking
areas in public places, and that this should be enough. To my mind, however, non-smokers should also be free
to go anywhere they choose without risking their health.
Smoking is harmful not just to smokers but to non-smokers too. If some people are foolish enough to continue
this dangerous habits, it seems to me that they should at least be prevented (10) from doing so in public.
1. a. Survey b. Conducts c. Statistics d. Evidence
2. a. damage b. destroy c. spoil d. hurt
3. a. thanks to b. in spite of c. because of d. due to
4. a. they b. these c. those d. things
5. a. definite b. indefinite c. indefinitely d. definitely
6. a. sit b. to sit c. sitting d. sat
7. a. add b. adding c. to add d. added
8. a. for b. as c. like d. such
9. a. protest b. object c. refuse d. deserve
10. a. for b. at c. from d. in
IV. Fill in each gap in the passage below with one suitable word.
For over a hundred years, the (1) people of London have gone to bed and (2) got up in the morning to the deep
sounds of the world (3) famous bell called Big Ben. The (4) bell appeared in Westminster Tower a year before
the clock, in 1858. It (5) weighs 13.5 tons, and it is the (6) largest bell in all England.
As we have (7) mentioned the clock that the bell serves appeared one year later, in 1859. Each of the clock’s
four (8) faces is almost 6.5 metres (9) in deameter. The munute hand is over 4 metres (10) long and weighs
over 100 kilograms.
(11) Although the clock is so big, it is very exact, it is seldom more than one second (12) wrong in 1924 hours.
English scientists use an original method to correct (13) mistake in the clock. There is a small tray in the middle
of the pendulum.If the clock is running slow, they put a penny coin in the tray, and the clock begins to run (14)
faster If the clock begins to run fast, they take (15) away the penny. The little coin is enough to make the giant
clock run a second faster or slower every day.
V. Use the word to make sentences.
Dear Betty
1. Thank you / invite me / your birthday party.
Thank you for inviting me to your birthday party.
2. I / afraid / not be able / come
I’m afraid I’m not able to come
3. We / have / many things to do / this week
We have got many things to do this week
4. The boss / ask / me / work overtime
The boss asked me to work overtime
5. I / promise / do so / and / have to keep / promise
I promise to do so and have to keep my promise
6. I / wish / can go to party
I wish I coould go to the party
7. If I / be there / all of us / have good time
If I were there, all of us would have good time
8. I’ll be thinking / you / when I / type / piles of letters
I’ll be thinking of you when I type these piles of letters
9. I / hope / your party / a success
I hope your party will be a success
10. I / wish happy returns
I wish you many happy returns.
VI. Finish each of the following sentences in such a way that it means the same as the one printed before it.
1. He started investigating the case a week ago.
He has been investigating the case for a week.
2. I’m quite sure that she didn’t steal the necklace.
She can’t have stolen the necklace.
3. The flat’s very noisy but we enjoy living there/
Even though the flat’s very noisy we enjoy living here.
4. “If I were you I wouldn’t trust Peter”, she told John.
She advised John not to trust Peter.
5. She is a far serious student now than she used to be.
She studies far more seriously now than she used to.
6. Your car does not give as much trouble as mine.
Your car gives less trouble than mine.
COMPOSITION
Smoking damages our health
(Smoking is harmful to our health)
(Việc hút thuốc có hại đến sức khoẻ của chúng ta)
Over the last thirty years, thanks to the efforts of various branches of science and technology, research has
proved that smoking damages health and shortens man’s life-span.
According to a diagram released by the World’s Health Organization (WHO), the number of people who die of
diseases connected to smoking is increasing more and more. The more science and technology learn, the more
damage caused by smoking is discovered. Therefore, it seems that we can never put a final stop to the list of
harm caused by smoking.
First of all, smoking brings about the decline of our memory because each cigarette we smoke destroys 15,000
cerebral cells.
Smoking is considered the number one cause of cardio-vascular trouble, pneumonia, lung cancer etc … The
main offending agents are carbon monoxide and nicotine in the smoke. These cancer – causing agents are also
the ones which cause pharyngeal cancer. The toxic effects of smoke make the smoker’s voice hoarse.
Additionally, the visible effect is that the tobacco or opium tar sticks to the teeth, and causes them to become
dark or dirty-yellow, diminishing our good looks and charm.
In conclusion, I would like to quote our well-known scholar “Le Quy Don” as saying some hundred years ago
that smoking was like taking a toxic drug, the smoke of which wasted our health and diminished our life-span;
yet, few people heeded his sage advice.
TEST 15
LEVEL: PRE-INTERMEDIATE & INTERMEDIATE
Cảm ơn bạn: Anh Phuong babephuong@gmail. Com đã đánh máy bài viết này:
I. Choose the word or phrase which best completes each sentence.
1. He’s just arrived to find his wife in tears.
a. embarrassed c. crying
b. panicking d. confused
2. She’s just bought a pair of silk tights.
a. pantyhose c. trousers
b. gloves d. socks
3. I am scared of living on my own in a big city.
a. tired c. bored
b. frightened d. fond
4. I feel sorry for those who have to do that kind of work.
a. pity c. apologize
b. excuse d. like
5. He was living in France when the war broke out.
a. ended c. occurred suddenly
b. burnt out d. spread
6. The land and the house that you own are your …………
a. property c. movable possessions
b. personal belongings d. savings
7. ………… are people who leave their country to live in another one.
a. Deserters c. Emigrants
b. Travelers d. Explorers
8. When you buy …………... in a company, you buy a part of that company.
a. staff c. food
b. machines d. stock
9. People ……….. their money because they want their money to grow in value.
a. save c. invest
b. put away d. hide
10. A(n) …………. is a person who sees something and can tell what he or she sees.
a. witness c. warden
b. observer d. newsreader
11. ………….. work is work which is done the same way all the time.
a. Routine c. Manual
b. Mental d. Office
12. When you ………. you can draw, write, compose music, or make something new.
a. make up c. discover
b. create d. imagine
13. Movies, sports, and reading are forms of ………… They help us relax.
a. exercise c. entertainment
b. study d. research
14. When a man …………. he asks a woman to marry him.
a. proposes c. engages
b. suggests d. offers himself
15. To …………means to helps someone remember.
a. recite c. memorize
b. remind d. reconsider
16. When a man ……….. a woman at her house, he goes to her house to get her.
a. drops c. drops off
b. picks up d. greets
17. A………… is an object that helps you remember a place you have visited
a. memory c. souvenir
b. diary d. note
18. When you buy tickets for the concert ………… you buy them before the time and day of the concert.
a. in advance c. by advance
b. in before d. ahead
19. People have a ………….. for special occasions, such as a wedding, a funeral, and a graduation.
a. meal c. dance
b. festival d. ceremony
20. When our friends have bad fortune, we try to show……………
a. love c. embarrassment
b. sympathy d. pity
II. Read the passage and then choose one answer for each of the following questions:
During the teenage years, many young people can be difficult to talk to. They often seem to dislike being
questioned. They may seem unwilling to talk about their work in school. This is a normal development at this
age, though it can be very hard for parents to understand. It is part of becoming independent of teenagers trying
to be adult while they are still growing up. Young people are usually more willing to talk if they believe that
questions are asked out of real interest and not because people are trying to check up on them.
Parents should do their best to talk to their sons and daughters about schoolwork and future plan but
should not push them to talk if they don’t want to. Parents should also watch for the danger signs: some young
people in trying to be adult may experiment with sex, drugs, alcohol or smoking. Parents need to watch for any
sign of unusual behavior which may be connected with these and get help if necessary.
1. This passage is taken from a
a. handbook for parents
b. school timetable
c. teenage magazine
d. book for children
2. Why do adults sometimes find teenagers difficult to talk to?
a. because most teenagers are quiet.
b. because teenagers don’t want to talk to other people.
c. because teenagers think adults are not honest.
d. because most teenagers hate adults.
3. When can you expect young people to be more talkative than usual?
a. When people talk to them because they are really interested and not just checking on them.
b. When adults give them a lot of money to spend.
c. When adults talk to them about something other than their work in school.
d. When adults talk to them about sex, alcohol, and drugs.
4. Some teenagers experiment with drinking and smoking because
a. cigarettes and alcohol are available everywhere.
b. cigarettes and alcohol are cheap.
c. women like smoking and drinking men.
d. they regard them as a mark of adulthood.
5. The word BEHAVIOR in the passage most nearly means
a. feeling.
b. teenagers.
c. activities.
d. reactions.
III. Read the article below and put a cross on the letter next to the word that best fits each space.
The last one?
After reading an article (1)…………. “Cigarette smoking and your health” I (2)………….. a cigarette to
calm my (3)…………… I smoke with concentration and pleasure as I was sure that this would by my last
cigarette. For a whole week I did not smoke at all and during this time, my wife suffered terribly. I had all the
usual symptoms of someone giving up smoking: a bad temper and an (4)……………….. appetite. My friends
kept on offering me cigarettes and cigars. They made no effort to hide their amusement whenever I produced a
packet of sweets from my pocket. After seven days of this I went to a party. Everybody (5)……………… me
was smoking and I felt (6)…………. uncomfortable. When my old friend Brian (7)………………… me to
accept a cigarette, it was more than I could bear. I (8)………………… one guiltily, lit it and smoked with
satisfaction. My wife was delighted that things had returned to normal once more. Anyway, as Brian pointed
out, it is the easiest thing in the world to (9)…………………. up smoking. He himself has done it
(10)……………..of time!
1. a. is b. entitled c. has d. made
2. a. fired b. lit c. burned d. ignited
3. a. nerves b. system c. brain d. head
4. a. small b. tiny c. enormous d. giant
5. a. over b. under c. above d. around
6. a. extremely b. not c. no d. none
7. a. prohibited b. urged c. allowed d. permitted
8. a. take b. took c. takes d. taken
9. a. quit b. abandon c. give d. stop
10. a. many b. some c. lot d. lots
IV. Fill in each gap in the passage below with one suitable word.
We need peace
No other word in the world is so (1)…dear….. and so (2)…near.... to the (3)…hearts…..of all the
common people on earth as the world (4)…peace… (5)…War….has always been harrible peace has always
been (6)…desirable…
We live in the atomic (7)…age....and the modern science (8)…achievements…..must serve peaceful
(9)…purposes….. The use of scientific knowledge for developing newer and more horrible (10)…means… of
destruction, especially the invention of the atom and hydrogen bombs has made the problem of peace and war a
matter of great urgency. It is quite clear (11)…that…. a war fought with such (12)…weapons….. would lead to
vast destruction in the (13)…world…. That is why the (14)…struggle…. For peace has become an urgent
necessity in our time.
V. Use the words to make sentences.
1. All / us took / holiday except Tom.
…………………………………………
All of us took a holiday except Tom.
2. We hold / stocks / cater / your requirements.
…………………………………………………….
We hold large stocks to cater for your requirements.
3. It take / me / long time / make / mind.
……………………………………………………
It takes me a long time to make up my mind.
4. English courses / will not / too hard / you study.
……………………………………………………….
The English course will not be too hard for you to study
5. Will not / committee decide / proposal today?
……………………………………………………….
Won’t the committee decide on the proposal today?
6. We must all / pressure / authorities / if / we / want / something / do/
………………………………………………………………………………
We must all put pressure on the authorities if we want something to be done.
7. He write / only correctly / but / neatly.
……………………………………………………..
He writes not only correctly but also neatly.
8. You said / your letter / you / like / visit us / summer.
…………………………………………………………….
You said in your letter that you’d like to visit us in the summer.
9. only can / computer gather / fact / also store.
……………………………………………………
Not only can the computer gather facts, it can also store them.
10. She play / only / guitar / also / violin.
……………………………………………..
She plays not only the guitar but also the violin.
VI. Finish each of the following sentences in such a way that it means the same as the one printed before
it.
1. She and I have never been there before.
Neither…… of us has been there before ………….
2. It was such good weather that we went swimming.
The weather…… was so good that we went swimming …………..
3. I can’t see that far.
It is too…… far for me to see ……
4. I have never been to the ballet before.
It is…… the first time I went to the ballet ……..
5. John doesn’t always speak the truth, I’m afraid.
You can’t…… believe John because he never speaks the truth ….
6. “Why don’t you put your luggage under the seat?” He asked.
He suggested…… putting my luggage under the seat ……………..
COMPOSITION
Your English teacher
ĐÁP ÁN:
I.
1.c 5.c 9.c 13. c 17.c
2.a 6.a 10.a 14. a 18.a
3. b 7. c 11. a 15. b 19. d
4. a 8. d 12. b 16. b 20. b
II.
1.a 2.c 3.a 4. d 5.b
III.
1.b 3.a 5. d 7. b 9. c
2.b 4.c 6. a 8. b 10. d
IV.
1. dear 6. desirable 11. that
2. near 7. age 12. weapons
3. hearts 8. achievements 13. world
4. peace 9. purposes 14. struggle
5. war 10. means
V.
1. All of us took a holiday except Tom.
2. We hold large stocks to cater for your requirements.
3. It takes me a long time to make up my mind.
4. The English course will not be too hard for you to study
5. Won’t the committee decide on the proposal today?
6. We must all put pressure on the authorities if we want something to be done.
7. He writes not only correctly but also neatly.
8. You said in your letter that you’d like to visit us in the summer.
9. Not only can the computer gather facts, it can also store them.
10. She plays not only the guitar but also the violin.
VI.
1. Neither of us have been there before.
2. The weather was so good that we went swimming.
3. It’s too far for me to see.
4. It is the first time I went to the ballet.
5. You can’t believe John because he never speaks the truth.
6. He suggested putting my luggage under the seat.
Your English teacher
My English teacher is Miss Loan. She’s of medium height, about twenty five of age. She’s fairy pretty
with her oval-shaped face, straight nose, large forehead and bright eyes. Though young and attractive in
appearance she’s dignified and virtuous.
She’s known to be a teacher of considerable ability. Her method of teaching differs somewhat with that of
our former teacher. Instead of explaining fully every new lesson beforehand she just gives us assignments and
let us work out ourselves. Thus her method of teaching corresponds to all of us and help us become active in
our English learning.
When teaching, she particularly pays close attention to English accent and pronunciation. She often
makes us pronounce an English word several times until it’s pronounced correctly and accurately. She always
encourages us in our English study and thanks to her devotion to teaching us, we have been making a rapid
progress.
Everyone in our class is now talking about our new English teacher who deserves our love and respect.
TEST 16
Cảm ơn bạn: "thuyhan tran" <thuyhan90@gmail.com> đã đánh máy bài viết này
I. Choose the word or phrase which best completes each sentence.
1. he was sacked having an argument with his boss.
a . hit
b. field
c. scolded
d. threatened
2. I was lucky enough to find a parking …..outside the bank
a. region
b. field
c. space
d. slot
3. The dustbins are …. every Thursday a. thrown
b. thrown away
c. space
d. emptied
4. I promptly went to sheep again
a. early
b. soon
c. immediately
d. after a white
5. English is required for that job
a. compulsory
b. interesting
c. extra
d. optional
6. Central heaving has been put in
a. received
b. installed
c. corrected
d. altered
7. Mary want to go New York by herself, but her father would not give his …..
a. control
b. example
c. consent
d. altered
8. Clara wanted to buy the coat, but it cost more than she could …
a. assume
b. afford
c. arouse
d. limit
9. The show should have stared an huor ago, I don’t know what could have caused …
a. alarm
b. delay
c. statement
d. custom
10. I sometimes take John’s coat instead of my own, because the two of them look so …
a. original
b. similar
c. comfortable
d. the same
11. George had difficulty swimming across the lake, but he finally succeeded on his fourth…..
a. attempt
b. process
c. display
d. instance
12. I know I have seen that man before,I can’t … where
a. assume
b. wonder
c. recognize
d. recall
13. I want to go the library, but I’m not walking in the right …
a. distance
b. circumstance
c. attitude
d. direction
14. I want to learn more about tha American political system, but I don’t know where to get the …
a. situation
b. information
c. conversation
d. association
15. Inthis hot weather the ice will soon…
a. spoil
b. melt
c. bake
d. fail
16. We had hoped that Robert would agree to help us, but he has …..to
a. desired
b. promised
c. esfused
d. intended
17. Since I have been ill, my appetite has diminished
a. desire for exercise
b. desire for sleep
c. desire for visitors
d. desire for food
18. Her husband is very competent ; he will repair the roof himsefl
a. talking about
b. thinking about
c. arguing about
d. putting off
19. The time for discussing the problem is over ; now we must act
a. talking about
b. thinking about
c. arguing about
d. putting off
20. I new my father would discipline me for my action
a. reward
b. reprove
c. congratualate
d. punish
II. Read the passage and the choose one answer for each of the following questions:
In many old cities in Europe, there are narrow twisting roads with many shops huddling together
along the two sides, These commerrcial places are not so modern and convenient as those called shopping
centers in modern cities, especially in the suburbs of the big cities in the United States.
Shopping centers have developed rapidly because of the shift og the population to the suburbs , the
growing use of and dependence upon the automobile and the heavy tracffic in downtown areas
A shopping center is a large group of stores facing a huge central enclosed mall which may be
covered , heated and air-conditioned .A shopping enter is olso surrounded by a parking area with space for
thousands of cars
We can buy on kinds of food and get anything we need in a chiefly sold a shopping center .Unlike a
supermaket, where groceries are chiefly sold , a shopping center provides us with all services besides food .We
canget our hair cut , eyes examined , clothes washed ,We can book our tickets for a would tour and even enroll
in special classes
Shopping centers are , therefore , very convenient for customers , but they lack the sense of closeness
as felt in older commercial center
1. The rapid development of shopping centers in mainly due to
a. the fact growing prosperity of suburban paople
b. the increased use of the automobile
c. the growing use of heavy cars in big cities
d. the shifl of the population to downtown areas
2. A shopping center is alarge group of stores facing a huge centrall mall which is
a. narrow and winding
b. very crowded with automobiles
c. used as a store-house for heaters and conditioners
d. shaded and com fortable
3. American shopping centers are especially established in the suburbs because
a. the customers want to avoid the heavy traffic in downtown areas.
b. the traffic is heavier in the suburbs than in the downtown areas.
c. there are few people moving from the downtown areas to the neighboring regions.
d. the streets in the downtown areas are so narrow and twisting.
4. Customers cant’t find the ‘sense of closeness’ in a modern shopping center because.
a. all the items in the stores are very expensive.
b. the shopkeepers are not very cordial.
c. it is too modern and conventional.
d. they worry too much about the safety of their cars.
5. In the shopping sections of many old cities in Europe, the stores are located.
a. in the auburbs.
b. in residential areas.
c. along poor, dirty roads.
d. along small, winding streets.
III. Read the article below and put a cross on the letter next to the word that best fits each space.
There has been a revolution in the world of newspapers. Not many years (1)……….., newspapers were
still being produced using techniques unchanged for (2)……….. hundred years.
The journalists gave their stories to a typist, who prepared them for an editor, who passed them on
(3)……….. the printer. The printer, who was a (4)……….. skilled man, set up the type. (5)……….. was then
collected to make the papes. When the pages were complete, the printing machines could be (6)………..
Nowadays what (7)………..? the journalists type their stories into a computer. The (8)……….. checks
their spelling, plans the page, shapes the articles. When the pages are ready, another computer may control the
printing.
(9)………..can be no doubt about it, producing a newspapers is an entinely different (10)……….. now.
1 a. before b. after c. ago d. yet
2 a. a b. some c. an d. over
3 a. to b. by a. through d. with
4 a. hardly b. mostly c. partly d. very
5 a. they b. which c. this d. all
6 a. switched b. started c. stopped d. moved
7 a. gives b. occurs c. goes d. happens
8 a. computer b. editor c. typist d. printer
9 a. it b. there c. you d. we
10 a. skill b. work c. management d. busuness
IV. Fill in each gap in the passage below with one suitable word.
In questions of preserving peace and saving mankind (1)……….. the threat of (2)……….. war, let one
remain (3)……….. or stand (4)……….. This concerns all and everyone. Each state is large or (5)………..,
socialist or (6)……….., has an important (7)……….. to make. Every responsible political party, every public
organization and every person can also make an important contribution.
No task is more (8)……….., more noble or humane than that of uniting all (9)……….. to achieve this
lofty (10)……….. This (11)……….. must be (12)……….. by our generation, not shifted onto the shoulders of
those (13)……….. will succeed us. This is the imperative of our time. This, I would say, is the (14)……….. of
historic responsibility (15)……….. our decision and actions in the time remaining until the beginning of the
third millennium.
Mikhail Gorbachev
V.Use the words to make sentences.
1. None/ us took/ holiday besides Tom.
………………………………………
2. Mr. Jones/ cut himself/ shave. ………………………………………
3. Please advise us/ soon/ goods have/ dispatched.
………………………………………
4. I see/ your advertisement/ today’s “Cambridge News”
………………………………………
5. I neither/ smoke/ drink.
………………………………………
6. I arrive/ Monday 4th
/ expected/ return/ London/ Thursday/ 7th
July.
………………………………………
7. No problem/ harder/ solve/ one.
………………………………………
8. The phone/ not be/ instead / I/ write/ you/ instead.
………………………………………
9. None/ her/ friend/ wished her/ happy birthday.
………………………………………
10. None/ students/ have/ finish/ exam yet.
………………………………………
VI. Finish each of the following sentences in such a way that it means the same as the one printed
before it.
1.Given fair warning, I could have avoided that date.
If you had told me …………………..
2. Please don’t say things like that.
I wish …………………..
3. It would have been a superb weekend if it hadn’t been for weather.
But …………………..
4. Dogs can swim better than cats can.
Cats cannot …………………..
5. She hasn’t been to the hairdresser for three months.
It’s …………………..
6. There’s a spare bed in David’s room.
David’s room …………………..
ĐÁP ÁN
I.
1.b 5.a 9.b 13.d 17.d
2.c 6.b 10.b 14.b 18.a
3.d 7.c 11.a 15.b 19.a
4.c 8.b 12. d 16.c 20.d
II.
1.b 2.d 3.a 4.c 5.d
III.
1.c 3.a 5.c 7.d 9.b
2.a 4.d 6.b 8.a 10.d
IV.
1.from 6.capitalist 11. task
2. nuclear 7. contribution 12. accomplished
3. indifferent 8. urgent 13. who
4. aloof 9. efforts 14. burden
5. small 10. goal 15. for
V.
1. None of us took a holiday besides Tom
2. Mr. Jone cut himsefl shavin
3. Please advise us as soon as the goods have been despatched
4. I’ve seen (saw) your advertisement in today’s “ Cambridge News”
5. I neither smoke nor drink
6. I arrived on Monday 4th
and expected to return to London on Thursday 7th
July
7. No problem is harder to solve than this one
8. The phone hasn’t been installed yet, do I’m writing to you instead
9. None of her friends wished her a happy birthday
10. None of the students have finished the exam yet
VI.
1. If you had told me in advance/ in (good) time I could have avoided that date.
2. T wish you wouldn’t say things like that.
3. But for the weather, it would have been a superd weekend.
4. Cat’s cannot swim as dogs (can).
5. It’s three months since she last went to the hairdresser.
6. David’s room has a spare bed in it.
Describe the career you have choden, and the reasons for your choice.
Choosing a career is of the most improtance. It determines one’s whole life. As for me, after careful
consideration, I want to become a construction engineer.
In our country, after long years of atrocious war, many houses, villages, bridges and roads were
destroyed by bombing. They all need to be repaired or rebuilt. After graduation, I’ll contribute my part to the
reconstruction of my war torn country into one of the most modern and powerful countries in the world.
How happy I feel when I am able to turn slum areas into blocks of hight buildings, to bring happiness
and joys to my poor compatriots!
How proud and delighted I am go to build high-ways, bridges and factorries for my city dwellers by the
sweat of my brow!
Cảm ơn bạn: "Thu Vuong" vuongthu90@gmail.com đã đánh máy bài viết này
TEST 18
LEVEL: PRE – INTERMEDIATE & INTERMEDIATE
I. Choose the word or phrase which best completes each sentence.
1. She said that she……….you some day.
a. had visited c. will visit
b. has visited d. would visit
2. In Prague it’s better to walk and………… the atmosphere of the pretty little streets.
a. feel c. think
b. tough d. move
3. Mr. Jones has…………… painting since he retired.
a. taken up c. taken over
b. taken of d. taken in
4. Most parents find it difficult to ………… their children nowadays.
a. grow up c. develop
b. foster d. bring up
5. When he was running across the road, he …………….a stone.
a. fell down c. fell off
b. fell over d. fell into
6. “I am absolutely broke”…………………….
a. So am I c. So do too
b. I do too d. I feel too
7. It was a light plane. It wasn’t………………
a. dark c. black
b. heavy d. deep
8. Often a team of engineers are………………
a. work on one project c. working on one project
b. on one project d. to working on one project
9. Please keep the fire………………….. It’s so cold in here.
a. burns c. to burn
b. burn d. burning
10. I’ve been living in this village since I…………………..
a. was born c. had born
b. were born d. have been born
11. Tommy admitted………………… the rock through the window.
a. throwing c. to throw
b. being throwing d. to be throw
12. How much dows that Swiss watch……………………?
a. pay c. spend
b. cost d. fix
13. ………………he has never borrowed any money from me.
a. Last week c. Since
b. Up to now d. A week ago
14. I……………..to inform you that your mather died ten minutes ago.
a. sorry c. apologies
b. regret d. pity
15. She was a young woman. The woman was………………….
a. a you c. a young
b. young d. youth
16. I can’t remember the lesson………………………
a. Either I can c. I don’t either
b.Neither I can d. Neither can I
17. I………………..Bob because I didn’t invite him to my party.
a. angry c. hit
b. offended d. missed
18. She wears…………..
a. an alarm clock c. a clock
b. an alarm d. a watch
19. Someone……………..my bicycle!
a. took c. takes
b. has taken d. is taking
20. we call the middle of the town the………………
a. town middle c. town capital
b. town heart d. town center
II. Read the passage and then choose one answer for each of the following questions:
The early expansion of the sugar industry was based on cane transported from two different
parts of the Orient, first from India and second from the islands of the Southwest pacific. From
India, sugar cane was carried through the western regions of Asia into Arabia and later into the
countries bordering on the Mediterranean. It was established in Sicily in about AD 703 and was
carried to spain in about AD 755. As early as AD 1150, Spain had at least 75,000 acres of cane.
During this early period sugar cane was grown largely to supply local wants. The Crusades were
partly responsible for the further expansion and improvement of the sugar industry and for
interesting Europeans in the use of sugar.
1. From this passage, sugar cane was first grown in
a. Europe c. Arabia
b. the Orient d. Sicily
2. Sugar cane was first introduced into Spain in
a. AD 703 c. AD 1150
b. AD 755 d. AD 1419
3. In the twelfth century, most cane was grown
a. for exploration.
b. for industrial uses.
c. for sales in the large market cities.
d. for local use.
4. One thing that caused the sugar industry to expand in Europe was
a. the favorable climate in the Orient.
b. the Crusades.
c. better means of transportation.
d. for local use.
5. Cane was transported
a. from the islands of the Southwest Pacific, to India, to Asia and then to Spain.
b. from the island of the Southwest Pacific, to the western areas of Asia and then to the
countries near the Mediterranean.
c. from India to the Southwest Pacific, to Asia, into Arabia and then to the countries bordering
the Mediterranean.
d. from India to Asia into Arabia and then to the countries on bordering the Mediterranean.
III. Read the article below and put a cross on the letter next to the word that best fits each
space.
It was a Monday morning in the office and, as usual everyone was busy sorting through their
mail before starting work. The other staff had (1)………… that Nicky, one of the computer
operators, had been (2)………….. rather miserable recently and they wondered what might have
happened to make her so (3)………… . Suddenly Nicky accidentally (4)………….. over a cup of
coffee. Asshe was clearing up the mess, the girl sitting next to her happened to glance at Nicky’s
right hand. There was a large swelling at the (5)…………… . “What on earth have you (6)
………….. to yourself? Does it (7) ………..?” asked the girl. “Yes, it does. I think you’d better go
and let the doctor (9)…………. It,” said the girl. “Let’s hope there’s no permanent
(10)……………. To your hand.”
1 a. recognized b. realized c. noticed d. identified
2 a. appearing b. seeming c. pretending d. looking
3 a. sick b. bored c. uninterested d. depressed
4 a. kicked b. punched c. hit d. knocked
5 a. wrist b. eblow c. knee d. ankle
6 a. made b. done c. happened d. brought
7 a. harm b. hurt c. disturb d. upset
8 a. injured b. ruined c. spoiled d. destroyed
9 a. check b. inspect c. examine d. investigate
10 a. pain b. destruction c. damage d. suffering
IV. Fill in each gap in the passage below with one suitable word. Write the words in the
spaces on your answer sheet.
My friend Jones is not a very practical person. (1)…………… along a main road one dark night
he suddenly had a flat (2)………….. Even worse he discovered that he did not have a
(3)……………….wheel in the back of his car Jones waved to passing cars and lorries, but no one
of them (4)…………….. Half an hour passed and he was almost (5)…………… despair. At
(6)……………… he waved to a car just like his own. To his surprise, the car actually stopped and
a well – dressed young woman got out. Jones was terribly (7)………….. How could a person like
this possibly help him? The lady, however, offered him her own spare wheel, but Jones had to
explain that he had never (8)…………… a wheel in his (9)……………She set to work at (10)
……………… and (11)……………..the wheel in a few (12)…………… while Jones looked on in
admiration.
V. Use the words to make sentences.
1. She/ secretly/ burned/ the letter/ fireplace
………………………………………………..
2. I’ll get him/ phone right.
……………………………………………….
3. I work/ contract/ this company.
………………………………………………
4. John should/ gone/ post office/ this morning.
……………………………………………….
5. The beat time/ call / be/ 6 p.m/ I / be always / home / that time.
…………………………………………….
6. Though he had / finished / paper / go/ sleep.
……………………………………………
7. I am / look / meet / you / airport.
…………………………………………….
8. She / very selfish woman / somehow you can’t / liking her.
……………………………………………
9. Turn left / Woodside Road and / the second / the right.
……………………………………………….
10.If she / seen / movie / she / have told you.
……………………………………………
VI. Finish each of the following sentences in such a way that it means the same as the one
printed before it.
1. I don’t really want them, but I’ll have them.
I might………………………….
2. He wishes he had invited her to his party.
He is sorry now……………..
3. They go out when the sun begins to rise.
They ……………………………
4. Is this the cheapest carpet you’ve got?
Have you got…………………
5. Nickel and silver do not differ greatly in appearance.
The difference…………………..
6. He doesn’t have enough money.
He’s …………………………….
ĐÁP ÁN
I.
1. d 5. b 9.d 13. b 17. b
2.a 6.a 10. a 14. b 18. d
3. a 7. b 11. a 15. b 19. b
4. d 8. c 12. b 16. d 20. d
II.
1. b 2. b 3. d 4. b 5. d
III.
1. c 3. d 5. a 7. b 9. c
2. d 4. d 6. b 8. a 10. c
IV.
1. driving 5. in 9. life
2. type 6. last 10. once
3. spare 7. disappointed 11. fitted
4. stopped 8. changed 12. minutes
V.
1. she secretly burned the letter in the fireplace.
2. I’ll get him to the phone right away.
3. I work on contract with this company.
4. John should have gone to the post office this morning
5. The best time to call is at 6 p.m, since I am always at home by (at) that time
6. Though he had not finished the paper, he went to sleep
7. I am looking forward to meeting you at the airport.
8. She’s a very selfish woman, but somehow you can’t help liking her.
9. Turn left into Woodside Road and take the second on the right.
10. If she had seen the movie, she would have told you.
VI.
1. I might as well have them
2. He is sorry now that he didn’t invite her to his party.
3. they go out in the early morning
4. have you got a cheaper carpet than this?
5. the difference in appearance between nickel and silver is not great.
6. he’s short of money.
Write about a holiday you took. Give details.
My friend and I recently went on a four – day holiday to Nha Trang. It was wonderful to
escape the air and noise of Ho Hi Minh city.
We took the night train and we were gently lulled to sleep by the sound of the train and
the pleasant night air. We arrived in Nha Trang early the next morning and checked into the
Grand Hotel which is very near to the beach, restaurants and nightlife. The hotle offered all
kinds of accommodation from first class to economy. The staff were friendly and extremely
helpful to us. They recommended a few tours which we took and we were not disappointed
with. On the beach we ate fresh seafood such as boiled crabs, grilled lobsters and sanguine
oysters. Nha Trang was clean and quiet, making our holiday very peaceful, restful and
unforgettable.
TEST 28
Cảm ơn bạn: Hang Cao" caohang.vcu@gmail.com đã thực hiện bài đánh máy này:
Level : pre - intermediate& intermediate
1 choose the word or pharse which best completes each Sentence
1 some diseases……… quickly from one person to another.
a. move c spead
b.catch d get about
2 he went to factory ………….. check the progress of work
a. for the purpose c so far as
b. by reason of d in order to
3 why are you looking ………….. me ? she asked .
a. one c at
b.over d to
4 He likes to go for a holiday …….. his wife .
a. without c except
b. less d unless
5 I’m looking for ward to you ………….. party .
a. birthday’s c birthdays’
b. birthday d birthdays
6 we’re going on a picnic ………. tomorow .
a. the day after c next day after
b. the next after d after
7 He was not very pleased about …………… called an incompetent i diot.
a. had been c to be
b. was d be ing
8 the new factory chimney was ………. Than all the trees around it .
a. longe c Taller
b. Greater d deeper
9 Is the …………….bread for all the sandwiches we have to make ?
a. Correct c Plenty
b. enought d equal
10 The was …………….enought cake for all of us to have a very mall slice.
a. Just c really
b. quite d hardly
11 The police – office ask us what …………….doing.
a. we had c we did
b. would we d we were
12 Mr white ………….the company in the way he wanted to.
a.owned c ran
b. rule d commanded
13 I absolutely………..wich everything that has been said .
a.accept c admit
b. agree d approve
14 One ………of this job is that it is near where i live
a.goodness c pleasure
b. prefer d advantage
15 He has a very ………. Temper and often says things he regrets later.
a.angry c wram
b. quickly d fast
16 The finance Minister will be making a …………. To day a bout new raters of income tax.
a.talk c statement
b. notice d declaration
17 I have never …………….. any experience of living in the country .
a.had c done
b. wished d made
18 pari lies …………. The river seine .
a.over c next
b.one d at
19 Could you please tell me if you have any elictric typewrites ………? .
a.in stock c in supply
b.in order d in store
20 She was unable to warm her mother that she would be late because the telephone was ……… .
a.out of work c off duty
b.off work d uot of odder
II Read the passage and then choose one answer for each of the following questions:
An eliction year is one in wich all four numbers are evenly divisible by four (1944,1948 ect .) sice 1840,
america presidents elected in years ending in zero have been destined to die in office. William H.Harrison, the
man who servel the shortest term, die of pneumonia
Several week after his inauguration.
Abraham lincoln was on of four presidents who were assassinated.he was elected in 1860 and his untimely
death came just five years later.
James a Garfield, a former Union army general from Ohio, was short during his first year
In office (1881) by a man to whom he wouldn’t give a job.
While in his second term of office (1901), William Mckinley, another Ohioan, attended the pan
American exposition at Buffalo, new york.During the reception, he was assassinated while shaking hands with
some of the guests.
Three years after his election in 1920 warren G, Harding died in office .Althoungh it was never proved,many
believe he was poisoned .
Franklin D Roosevelt had been eclected four time (1932,1936,1940 and 1944), the only man to serve so
long a term.He had contracted polio in 1921 and die of the illness in 1945 .
John F Kennedy,the last of the line was assassinated in 1963 only three years after his eliction.
Will 1980’s candidate suffer the same fate ?
1 Which of the following was not an election year ?
a,1960 b,1930 c,1888 d,1824
2 which presidedent served the shortest term in office ?
a, Abraham lincoln b, William Mckinley
c, warren G, Harding d,William H harrison
3 Which of following is true ?
a, all presidents elected in years ending in zero have died in office .
b, Only presidents from Ohio have died in office.
c, Franklin D .Roosevelt complated four terms as president .
d, Four American presidents have been assassinated
4 How many presidents elected in years ending in zero sice 1840 have died in office ?
a,7 b,5 c,4 d,3
5 Which of the following was not assassinated ?
a, John F Kennedy c, Abraham lincoln
b, Franklin D Roosevelt d,Jame A, Grarfield
III Read the article below and put a cross on the letter next to the word that best fist each space
WATER
Warter is our life source .it make up 70 per cent ,of (1)…………. Bodies,and average person actually spends
18 months of his life (2) ………………… the bath of shower.
But we are only now leaning how to look (3) ………………water .Acis rain (4)..................polluted with
waste products .It is now (5)……………..expensive to try to repair the damage which has been done .We have
some hope for the future,though,(6)……………….new sources of water have been discovered.People
(7)………….in the sahara Derert have (8) ……………….fish swimming in deep underground streams.
Scientists also bilieve (9)…………………… is a huge lake underneath London.If we have
(10)…………… from our mistakes,we will try keep the new areas of water clean.
1a,the b their c our d these
2a in b to c on d at
3a over b on c after d to
4a will b has c would d is
5a very b such c more d much
6a which b because c so d even
7a live b living c to live d lived
8a realised b put c looked d found
9a it b here c that d there
10a taught b Practiced c Known d learnt
VI Fill in each gap in the passage below with one suiltable word.
Concorder, the world’s fattest and the (1) ………… graceful passenger plane, will soon be 25 years old,It fist
flew on 2 March 1969, from Toulouse in France .
Concorder(2)……….. developep (3) …………..France and Britain.From 1956 these (4)………..countries had
a dream of a supersonic passenger plane the plane (5)……………over 1,5 billion on develop.It is the most
tested plane in the ( 6)…………
Of aviation .It was given over 5,000 hour’s of testing.
Concorder flies at twice the speed of sound .This mans that it (7)…………………only 3 hours 25 minutes to
fly (8) ………………Lon don and New York,compared with 7-8 hours in other(9) …………………
Each Concorde is (10) ……………at a cost of 55 million.Twenty haven been built so far Air France and
British always own the most. they each have seven planes
V Use the word to make sentencens
Dear peter
1 I/feel/very happy /get/letter/after/such/longtime.
2 It /be /absolutely a year /since/i hear you .
3 You like /come/stay/me/countryside?
4 you be /very welcome /you want /visit.
5 What about /come end /next months ?
6 My wife /miss /you both /often speaks/you .
7 Don’t think /chirldren/love /visit /countryside?
8 Write soon /let /know/when /you /come
Fondly,alane
VI Finish each of the folloing sentences is such a way that it means the same as the one printed before it .
1 we must keep our school clean and tidy
Our school …………………………….
2 He stay at home .it rained hard
Because ………………………………..
3 They were selling tickets when the match started
Tickets …………………………………
4 They like fied events .They like team games
They like not only ……………………...
5 They were giving the captain some presents when we left the stadium.
The captain……………………………..
6 They are glad because they have a friend like him.
They are ……….to …………………………
Composition
Tell about your in door hobby and why you enjoy it
Đáp án
1 c 5b 9b 13b 17a
2 d 6a 10a 14d 18b
3 c 7d 11d 15b 19a
4 a 8c 12c 16c 20d
II
1.b 2.a 3c 4.a 5b
III
1c 3c 5a 7b 9d
2a 4.b 6.b 8d 10d
IV
1 most 5 takes 9 planes
2 Science 6 only 10 sold
3 flight 7 both
4 was 8 between
V
1 I fell very happy to get your letter after such a long time
2 If had been absolutely a year since i heard from you .
3 Would you to like to come and stay with me in the contryside ?
4 You are very wellcome if you want to visit me .
5 What about comming at the end of next mounth ?
6 My wife misses you both very much and often speaks about you .
7 Don’t you think your chirldren would love to visit the countryside .
8 Write soon and let me know when you can come
VI
1 Our school must be kept clean and tidy
2 Because it rained hard ,he stayed at home
3 Tickets were being sold when the macht started
4 They like not only filed events but also team games
5 The captain was being given some prensets when we left the stadium.
6 they are glad to have a friend like him
Viết đoạn văn
Tell me about your indoor hobby and why you enjoy it
Experience has taught me a valuable lesson,which is that i should have an in door hobby to occupy myself in
my spare – time, othe wise i might spend my lei sure – time on entertaining games which are either
mindless or wasteful. After careful consideration i decided to sart stamp collection since i regard postage
stamps as object of interest and value .For stamps to be accepted for use as postage, stamps undergo screening
steps , and only those stamps which excelled the others in meanings ,in artistic skills, and aesthetic designs will
be selected .So a stamp is really a cultural trait of the country which issues it .
What could be more interesting than having in my hand such a masterpiece?
Stamps teach leasons on geography and history. I, of course , will have to look up from dictionaries or
other books to discover mor about the origin of stamps. This will broaden my geographycal and historical
knowledge more and more
Then, collecting stamps is quite a relaxing activity too .I can well imagine how a new beautifuly
stamp, whether it is a domestic or overseas one,Cuold make me happy whatching it for hours on end so my
hobby can help me relax
All in all, stamp collecting further stimulates my creative and intellectual growth,wich makes me nevers feel
guilty about wasting time
TEST 26
Cảm ơn bạn: na na" nana.itp@gmail.com đã thực hiện bài đánh máy này:
I. Choice the word or phrase which best completes each sentence
1. Are there any good ……….. on television tonight
a. Shows c. Programmes
b. Screens d. Performances
2. There’s a very interesting ………… about life in the Arctic
a. news c. service
b. Documentary d. return
3. That’s a ………., isn’t it? I think I saw it about a fortnight ago
a. second c. copy
b. repeat d. return
4. It’s part of a ………… on living in strangge places
a. set c. series
b. collection d. group
5. Do you watch a lot of ………. Firm?
a. travel c. tour
b. journey d. sighting
6. I prefer television……………
a. stages c. acts
b. screens d. dramas
7. What about …………….. programmes?
a. playing c. sports
b. gaming d. match
8. I like to see “Match of the Day” on Saturdays. That’s my……………
a. popular c. preferable
b. favorite d. likeable
9. Is Cliff Lion still the………..?
a. Presenter c. actor
b. talker d. reader
10. Cliff Lion usually discusses the matches with two or three footballers in the……..
a. stage c. studio
b. theatre d. floor
11. Chas Merton has asked me to write a ……………… of the programme for the Center City Daily News.
a. judgement c. review
b. value d. paper
12. You’ll probably see Dave on the 4:30 bus, he………… catches it to school
a. sometimes c. routinely
b. traditionally d. usually
13. The doctor told me to take the pills ……….. three times a day
a. invariably c. customarily
b. regularly d. practically
14. Timothy is …………… late when he meets Marble
a. frequently c. customarily
b. routinely d.practically
15. People who …………… tell lies should be punished severely
a. traditionally c. practically
b. habittually d. customarily
16. Would you believe it! It………. Rains for this particular festival!
a. inconsistenly c. invariably
b. cuatomarily d. changelessly
17. Linda…………… scores very high marks in grammar test but quite low marks in composition writing
a. traditionally c. customarily
b. consistently d. routinely
18. Can you tell me what time the doctor’s ………….. open?
a. office c. hospital
b. surgery d. ward
19. Is it for some ………… for a headache?
a. improvement c. liquid
b. prevention d. medicine
20. This should soon …………….. your cough
a. prevent c. disappear
b. solve d. cure
II. Read the pasage and then choose one answer for each of the following question:
An Enlish man living near Sheffield had a most unlucky day’s fishing. He was standing on a law bridge when
he had the misfortune to knock his tin of bait into the river. Learning over the side of the bridge, he tried to
hook the tin and pull it out of the river. As he did so, his car keys fell out of his pocket and disappeared in the
water.
Feeling thoroughly annoyed with himself and his luck, the fisherman lent over the bridge to try to see
where his keys had gone. As he đi so, so bridge collapsed and he fell into the river. This was the last straw. The
fisher crawled out of the river and went back to his car. There he discovered that he had locked the doors and
colud not get in. His anti-thief had immobilized his car. There was nothing left for him to do but walk sadly
home.
1. How did the tin of bait get into the river?
a. The passage does not say
b. The fisherman knocked it in accidentally.
c. It was probably blown in the wind
d. It fell in when the fisherman fell in.
2. When did the fisherman fist lean over the bridge?
a. when it broke
b. when the tin of bait fell in
c. when his key fell in
d. a few minutes after he first started fishing
3. What was the last straw, according to the passage.
a. having to walk home
b. being unable to get into his car
c. the loss of his car keys
d. the collapse of the bridge
4. What is the meaning of “misfortune”?
a. bad luck c. accidentally
b. good luck d. a clumsy act
5. What is the meaning of “thoroughly”?
a. slightly c. through
b. at last d. extremely
III. Read the article below and put a cross on the letter next to the word that best fits each space
From the moment they have the security of their accustomed environment, travelers are at risk. Environment
factors such as a arduous condition, (1)…….. climate, and high altitude may cause a danger, and so may
travelers’ own behavior, free from the (2)…….. of the daily routine, and determined to have a good time with
little (3)………. For the consequences.
When illness or injury occur abroad, travelers are again at a disadvantage from (4) ……… to communicate
with a doctor on account of language and cultural difficulties or being unable to find a doctor owing to
(5)…….. of the local medical system. There may be a complete (6)……….. of skilled medical care, or of
medical facilities of (7)……… acceptable to travelers from technologically developed (8)………
1. a. worsening b. unus c. sudden d. unfavorable
2. a. restraints b. assurances c. certainties d. regulation
3. a. knowledge b. awareness c. regard d. need
4. a. inability b. difficulty c. inflexibility d. timidity
5. a. miser b. doubt c. ignorance d. disbelief
6. a. breakdown b. failure c. disruption d. absence
7. a. type b. design c. standard d. degree
8. a. countries b. places c. areas d. locations
IV. Fill in each gap in the passage below with one suitable word.
I do my best to take care of my (1) …………. And my family’s but it’s not always easy. My wife and I (2)
……….. the cooking and shopping and I’m a bit more careful than she is not about buying foods that I don’t
have colorings or other sort of (3) ……………. In. I know you can’t (4)……….. things like that entirely, but I
really try to do my best. I think that chemical probably (5) …………… more health problems that most people
think they do. We don’t (6)……….. and in fact don’t have many friends who do, but my mother can’t live
without her cigarettes, and of course when she is here all of the air we (7)………… has some smoke in it.
The whole family gét some kind of (8) …………….. both the kids are very keen on swimmers, my wife
runs (9)……… and I play tennis at least three times a week.
Acctually I think we are (10)………….. now then when we (11) ………… younger, we began (12)
…….more attention to food when the kids (13) ………… young and wanted to give them a good (14)
…………. By keeping fit ourselves. I certainly get fewer (15)………. Colds and flu than I did then.
V. Use the words to make sentences
1. Arrange/ kitchen wares as/ mother/ do
2. Our company/ well-know/ whole saler/ ceramics.
3. She dances/ much/ beautiful/ than predecessor
4. did not you enjoy/ see her after/ many/ year?
5. The chair/ too big/ put/ that room
6. How does/ mary/ life agree/ you?
7. You/ looking/ lovely/ ever.
8. He/ only play/ piano/ also compose/ music
9. Because/ roughnessd/ road/ car would go/ furture
10. You can take / main road/ come/ crossroads/ station.
VI. Finish each of the following sentences in such a way that it means the same as the one printed before
it.
1. Mary told the police about the burglary
Mary reported………………
2. I never intented to go to the wedding.
I never had……………………
3. “It certainly wasn’t me took your car” said Bob
Bob reused……………………………..
4. Simon had no expected that he would feel so weak after the opẻation.
The operation left……………..
5. Although he was not guilty they executed him
In spite of………………………….
6. He was suspended for two matches for swearing at the referee
Swearing at the referee earned……………….
Composition: you have just arrived to visit a foreign country. Write to your parent announcing that you
have arrived safely and ask them to send something to you
Đáp án test 26
I.
1c 5a 9a 13b 17b
2b 6d 10c 14a 18b
3b 7c 11c 15b 19d
4c 8b 12d 16c 20d
II.
1b 2b 3d 4a 5d
III.
1d
2d
3c
4a
5c
6d
7c
8a
IV.
1. health
2.do
3.chemicals
4. avoid
5. cause
6. have
7. breathe
8 sports
9. regularly
10. healthier
11. were
12. paying
13 were
14 health
15. sickness
V.
1. Arrange the kitchen wares as your mother does
2. Our company is well known as a whole saler of ceramics
3. She dances much more beautifully than her predecessor
4. Didn’t you enjoy seeing her after so many years?
5. The chair is too big to put in that room
6. How does married life agree with you?
7. You’re looking lovelier than ever
8. He not only plays the piano but also composes music
9. Because of the roughness of the road, the car would not go further
10. You take the main road until you come to the crossroads near the station
Composition:
Dear mother and father,
This letter is to let you know that I have arrived safely in Singapore. My first impressions are that Singapore
is a beautiful country and that I am going to enjoy staying here very much.
Everything is so clean and beautiful and very well- organized. The airport is very large and modern and I
could see that they have just completed the modernization. The people working there were friendly, courteous
and very helpful. They made me feel very welcome to their city. The taxi from the airport downtown was
readily available and very cheap.
My hotel is new modern one with every convenience. The tourist bulletin that I found in my room suggests to
me so many different things that I might like to see and do, so I am sure that I will be busy every minute.
Unfortunately, I forgot the shopping list that I had made out before I came here. As you know, many of my
friends asked me to buy something for them. I can not possibly remember everything, so would you please send
the list to me.
I miss both of you and wish that you were here with me. Don’t worry about me because this is a very secure
and safe place. I will write again in a few days.
Your loving son,
Cảm ơn bạn Mr Silver" <mrsilverwolf89@gmail.com đã đánh máy bài viết này:
TEST 30
Level: PRE-INTERMEDIATE & INTERMEDIATE
I/Choose the word or phrase which best completes each sentence:
1.It is usually better not to ……….. things, in case they are not returned.
a. lose c. borrow
b. offer d. lend
2. He stood on one leg, ………… against the wall, while he took off his shoe.
a. leaning c. stopping
b. bargain d. supporting
3. If we go to the market might find a …………. .
a. trade c. stopping
b. staying d. shopping
4. They haven’t beaten me yet. I still have one or two ………….. up my sleeve.
a. traps c. tricks
b. jokes d. defenses
5. My mother was …………. of making a cake when the front door bell rang.
a. at the center c. halfway through
b. on her way d. in te middle
6. …………., after trying three times, he passed the examination.
a. Lastly c. Last
b. last of all d. At last
7. The doctor told him to keep ………. Sweets and chocolate to lose weight. A. at c. off
b. back b. up
8. Nobody seems to be ………….. control of those children.
a. under c. with
b. in d. over
9. …………. Many times I tell him, he always forgets to pass on phone messages.
a. However c. Wherever
b. Whatever d. Whenever
10. This is ………….. the most difficult job I’ve ever had to do.
a. fells c. holds
b. touches d. by myself
11. This cloth ………… very thin.
a. by chance c. by heart
b. by far d. by myself
12. Will the government be able to …………. All their immense difficulties?
a. overcome c. dismiss
b. defeat d. overtake
13. My application for a trading license was ………….. .
a. held down c. put down
b. turned down d. let down
14.The opening ………….. of the play took place in an army camp.
a. stage c. scene
b. sight d. piece
15. I have told him never …………. here again.
a. come c. came
b. comes d. to come
16. As soon as the consumer protection law was passed, some manufacturers began to
…………. Have it changed.
A, object c. campaign
b. revolt d. protect
17. The BBC tries to ………….. for all tastes with its four national programmer.
a. cater c. regard
b. furnish d. suit
18. The tank of petrol was ………… by a carelessly discarded cigarette end.
a. ignited c. exploded
b. lit up d. inflamed
19. He gave me a furious look and …………… out of the room.
a. surged c. stormed
b. hurled d. burst
20. Unless the Prime Minister …………… the warning, inflation will rise rapidly.
a. remarks c. applies
b. attends d. heeds
II. Read the passage and then chose one answer for each of the following questions:
In 1972 there arrived in Philadelphia a penniless young man, eager for work and for
knowledge. As the year passed, this man, Benjamin Franklin, contributed greatly to hos city
and to his country. He became a printer and a publisher, and a learned man in many
subjects. Ha also helped to spread learning by establishing a public library and by founding
the American Philadelphia Society, which is an important academy of great scholars to this
day.
Franklin initiated many improvements in the city of Philadelphia, making one of the
world’s first cities to have paved and lighted streets as well as a police force and a fire
fighting company. He also made many practical inventions such as the Franklin stove, which
was a very efficient storms. His scientific work with electricity earned Franklin world fame.
Franklin played an important role in the early history of the United States. He took part
in drawing up the Declaration of Independence and the Constitution. He was the first
ambassador to France, and he helped negotiate the treaty of 1783, which ended the
Revolutionary War.
As an active member and as president of the Abolitionist Society, Franklin devoted the
last years of his life to the movement to end slavery.
1. When Franklin arrived in Philadelphia in 1723, he was
a. rich c. neither rich nor poor.
b. well-off d. very poor.
2. How did Franklin help to spread learning?
a. He was eager for work and knowledge.
b. He became a printer and a publisher.
c. He contributed to his city and his country.
d. He established a public library and founded the
American Philadelphia Society.
3. What made Franklin famous all over the world?
a. The Franklin stove.
b. The lightning rod.
c. The fire fighting company.
d. His scientific work with electricity.
4. Which of the following statements is not true?
a. Franklin alone drew up the Declaration of
Independence.
b. Franklin was one of those eminent Americans
who drew up the Declaration of Independence.
c. Franklin was the first American ambassador to
France.
d. Franklin helped negotiate the treaty of 1783.
5. In what was Franklin active during the last years of his life?
a. The Revolutionary war.
b. His scientific work.
c. His practical inventions.
d. The abolitionist movement.
III. Read the article below and put a cross on the letter next to the world that best
fits each space.
An Australian Mystery
Interest in undiscovered human-like creatures to be widespread. Every one has (1)
………….. of the Yeti and its North American “cousin” Bigfoot, but since the last century there
have been (2) ………….. of the existence in Australia of another, less famous creature-the
Yahoo. In 1912, a Sydney newspaper (3)………….. an account by Charles Harper of a
strange, large animal he observed (4) …………… the light of his campfire “It body, legs and
arms were covered with long, brownish-red hair, but what (5) ………….. me as most
extraordinary was its shape, which was human in some ways (6) ………….. at the same time
very different. The body was enormous (7) …………… great strength. The aims were
extremely long and very muscular”
Harper continued: “All this observation (8) ………….. a flew minutes while the creature
stood there, as if frightened by the firelight. After a flew growls and beating his breast, he (9)
………….. the first few metres upright, then on all four limbs through on the trip, a fact at
which I must admit I was rather pleased”
1. a. understood b. known c. heard d. noticed
2. a. statements b. reports c. arguments d. proofs
3. a. delivered b. typed c. declared d. printed
4. a. by b. at c. with d. under
5. a. marked b. struck c. touched d. knocked
6. a. even b. just c. still d. yet
7. a. announcing b. pointing c. indicating d. engaged
8. a. lasted b. covered c. involved d. engaged
9. a. set back b. set up c. set in d. set off
10. a. should b. must c. might d. would
IV. Fill in each gap in the passage below with one suitable word. Write the words in the
spaces on your answer sheet.
In a village on the East coast of Scotland, people were waiting anxiously for news.
Two of their fishing boat (1) ……….. been caught in the storm (2) ………… had blown up
during the night. In the cottages round the harbor people stood (3) ………… their doors (4)
………… worried to talk.
The rest of the fishing fleet had (5) …………. The harbor before dark and the men
from these ships waited and watched with the wives and families of (6) ………….. missing
men. Some had (7) …………. Thick blankets and some flasks of hot drinks, knowing that
the men (8) ………… be cold and tired. As dawn began to break over (9) ………… the
East, a small point of light was (10) …………. In the darkness of the water and a (11)
………….. minutes later, (12) …………. Was a shout. Before long, the two boats (13)
………….. turning in past the lighthouse, to the inside of the harbor. The men (14)
…………. helped out of their boats, and although they were stiff (15) ………… cold and
tiredness, they were all safe.
V. Use the words to make sentences.
Dear Sir
1. My wife / I / now / plan / holiday / next summer.
2. We / like / go / Italy / two weeks.
3. Holiday / begin / July 15th
/ we return / July 29th
4. My wife / dislike / travel / coach / we / wish / go / air.
5. We / expect / stay / hotel / east coast.
6. We / grateful / you / send / information / details / charges.
7. I / trust / you / able / help / arrangements.
Yours sincerely,
E. Smith
VI. Finish each of the following sentences in such a way that it means the same as
the one printed before it.
1. Then she asked “Who was the first president of the United States?”
She asked the student …………………………………………………………
2. The teacher asked “Do you understand my question?”
She asked the student ……………………………………………..
3. Then she said, “I didn’t tell you to go!”
She told the student …………………………………………….
4. The students answered, “I thought you asked the next student.”
The student answered ……………………………………………..
5. When John arrived home, he told his mother his decision.
Arriving ………………………………………..
6. After John had watched the garbage collectors on his street, he decided he wanted to
become one, too.
Watching ………………………………………….
COMPOSITION
Describe a day spent by the river or bi the sea.
TEST 30
I.
1. d 5. d 9. a 13. b 17. a
2. a 6. d 10. b 14. c 18. a
3. b 7. c 11. a 15. d 19. c
4. c 8. b 12. a 16. c 20. d
II.
1. d 2. d 3. d 4. a 5. d
III.
1. c 3. c 5. c 7.a 9.c
2. b 3. b 6. c 8. d 10. c
IV.
1. had 6. The 11. Flew
2. which / that 7. Brought 12. There
3. at / by 8. Would 13. Were
4. and 9. In 14. Were
5. reached 10. Seen 15. With / from
V.
1. My wife and I are now planning a holiday for next summer.
2. We’d like to go to Italy for two weeks.
3. Our holiday will begin on July 15th
and we’ll return on July 29th
.
4. My wife dislikes travelling by coach, so we wish to go by air.
5. We expect to stay at a hotel on the east coast
6. We’d be graterful if you sent us information with details of charges.
7. I trust you be able to help in these arrangements.
VI.
1. She asked the student who the first president of the United States was.
2. She asked the student if he understood her question.
3. She told the student that she hadn’t told him to go.
4. The student answered that he had thought the teacher had asked the next student.
5. Arriving home, John told his mother his decision.
6. Watching the garbage collectors on his street, he decided he wanted to become one,
too.
TEST 30
Describe a day spent by the river or bi the sea.
Summer comes together with its extreme heat and nobody can resist the temptation
to spend their holiday at the seaside.
Last week, in older to escape from the suffocating atmosphere of a crowded city, my
friends and I decided to go to Vung Tau beach. When we got to the seaside, the beach had
already been crammed with holiday makers. They were either sunbathing on the beach for a
short tie, we all took off our clothes and let ourselves drift along on the waves.
Swimming brought us a good appetite. We enjoyed some sea foods and then took a
quick nap after lunch
In the afternoon we strolled along the beach, inhaling in rapture the fresh sea air and
took lots of photographs.
A holiday at the seaside helped us get rid of all worries and trouble and gave us
health and enthusiasm to resume work again.
TEST 1
Cảm ơn bạn Hà Lê" thiha1989dhtn@gmail.com đã đánh máy bài viết này
LEVEL: PRE – INTERMEDIATE & INTERMEDIATE
I/ Choose the word or phrase which best completes each sentence.
1. A good clock always keeps…………… time.
a. Certain c. accurate
b. True d. serious
2. She has……………. a lot of money in her new job.
a. Gained c. found
b. Earned d. done
3. Would you……………. me opening the windows now?
a. Want c. concern
b. Worry d. mind
4. We can………… the difficulty without too much effort
a. Get over c. get off
b. Get away d. get through
5. ………… people go the movies now than ten years ago
a. Less c. Few
b. Lesser d. Fewer
6. We don’t know the …………….. of the game.
a. Facts c. rules
b. Customs d.laws
7. We had to ues our neighbor’s telephone because ours was………..
a. Out of work c. off duty
b. Out of order d. off work
8. It was ………… a boring speech that I felt asleep
a. Such c. very
b. So d. too
9. I think you…………… better not tell him truth.
a. Will c. would
b. Should d. had
10. What he says makes no ……………. to me
a. Reason c.sene
b. Truth d. matter
11. He said that he ……………badly sick since he returned from a broad.
a. Had been c. was
b. Has been d. would be
12. The speaker said …………… nothings worth listenning to.
a. Complete c.quite
b. Almost d. completely
13. If you had come to the party, you………………. Her
a. Would meet c. would have met
b. Had met d. met
14. The manager had his seretary…………. the report for him
a. Type c. typing
b. Typed d. to type
15. He………….. in this office for ten years by next Monday.
a. Will be working c. will have been working
b. Will work d. would working
16. No sooner ……………… the office than the phone rang.
a. He had left c. he was leaving
b. Had he left d. was he leaving
17. Neither the director nor his assistant …………… yet.
a. Have come c. has come
b. Haven’t com d. hasn’t com
18. It is necessary that you …………… here at 8 a.m. tomorrow.
a. Would be c. will be being
b. Be d. would have been
19. The village……………. We are going to visit is far from here
a. Where c. that
b. Which d. both b and c are correct
20. She wishes she …………….. you the bad news yesterday.
a. Didn’t tell c. hadn’t told
b. Woundn’t tell d. doesn’t tell
II/ Read the passage and then choose one answer for each of the following questions:
A gold rush is the rapid relocation of large numbers of people to an area where gold has been
discovered. Gold rushes capture the imagination and participation of many people because of the magical lure
of gold and the potential for overnight affluence. The greatest gold rush in united States history was the
California Gold Rush of 1849.
The rush first began in 1848 when a carpenter named James Marshall discovered gold on the property
of John A. Stutter in the Sacramento Valley. Hired to build a sawmill on the banks of the Amarican River,
Marshall had hardly begun work when he started finding nugget after nugget gold. News of the discovery at
Stuttet’s mill spread quickly, and soon thousands of persons were laying claims in the area. These people,
called “ forty – niners”, rushed in from all over the world. In just two years, the population of california
increased from about 26,000 to 380,000, Consequently, Californiawas officially admitted to the union as a
state, in Stemper of 1850. The free – spending style of the successful miners helped to turn communities such
as Sacramento and San Fancisco into prosperous towns. Those who were not so lucky became farmers and
rachers in the Central Valley of California.
1. what is the best title for this passage?
a. Famous gold Rushes.
b. The forty – nines.
c. The lure of gold
d. The california Gold Rush of 1849.
2. According to the passage, people join gold rushes because………………..
a. They are graranteed overnight wealth.
b. Gold hold a magical power for everyone.
c. They believe they have a chance of becoming affuent.
d. Thay have very active imaginations.
3. According to the passage, where was gold first discovered?
a. At Stutter’s mill.
b. On James Marshall’s property near Sacramento.
c. In the streambed of the American River.
d. In the central valley of california.
4. It can be inferred from the passage that the California gold rush…….
a. Provided most “ forty-nines” wih long – term mining work
b. Attracted people from every country in the world.
c. Spead throughout the Central Valley.
d. Greatly Speeded up the development of carlifornia.
5. According to the passage, what qualifire California to be admitted to the union?
a. The status of Statehood.
b. The great increase in population.
c. The political efforts of succassful miners.
d. The prosperity of California communities.
III/ Read the article bolow and put a cross on the letter next tho the word that best fits each space.
Shopping in the Mappstone is a must if you are visiting the area.
There (1) …………. Many shops and services all within walking distance (2) …………. The cetal square. The
city is particularly busy during (3) ……………… summer months of june, July and August when tourists visit
Mappstone from all over the (4) ………………One of the main streets of the central square – Cedar Avenue –
is the most popular shopping area for not (5) ………….. visittors but residents too. Stores line both sides of the
avenue and often (6) …………. Open late into the evenung. Most of the shops in this avenue are small and
expensive (7) …………. In nearby streets shoppers can find almost anything at more reasonable (8) ………….
Tuorists wiil find that many of these shops offer and export service and goods (9) ………….. be posted direct
tho the buyer’s home country. Shops ara usually closed on Mondays but ortherwise open from 9 am in the
morning (10) ……….. 8 pm in the evening.
1 a. is b. are c. have d. be
2 a. of b. in c. by d. to
3 a. some b. a c. the d. one
4 a. earth b. ground c. space d. world
5 a. same b. just c. exactly d. very
6 a. stay b. go c. make d. put
7 a. since b. so c. but d. because
8 a. can b. numbers c. values d. prices
9 a. can b. vould c. might d. should
10.a. with b. up c. until d. for
IV/ Fill in each gap in the passage lelow with one suitable word.
One of the greattest (1) …………for foreign students in American universities is the lecture system.
The professor (2) ………….and the students take (3) ……….. These notes are later used to study for (4)
…………….. .If the notes aren’t good, it will be (5) ………….. for the student to (6) ……….. for a subsequent
exam. It is not easy to take good notes from a lecture in a (7) …………. Language. You try to write every (8)
…………. The professor says, then you are writing at the same time the professor is lecturing, this kind of note
(9) means that you will fall far (10) ……….. the lecture, and the notes wiil be confused. They may be (11)
…………. For later study.
What can you do to take better notes? The first skill to learn is how to write only the most (12)
…………. Words, not whole sentences. (13) ………….. words are the ones that present new information. The
most important words are often the focus of intonation. They are usually emphssized by pith change, and this
should be your signal of (14) ………. Content words, like nouns and verds, are usually be focus of information,
so your notes should be almost entirely (15) …………. Words. Remember that you need information, not
sentences.
V/ Use the word to make sentences.
1. Please/not forget/put/names/on them.
2. although/they/brothers/they/not look/alike.
3. I/really/not want/dessert.
4. It/never/late/ go back/school.
5. Jeff/bought/a twenty dollars sweater/half price.
6. Study/all night/good/for/neither/your grades/nor/health.
7. we’ll/have/leave/eight o’clock/get/ office/before nine.
8. Mary/lost/ a lot/ weight/ her diet
9. I/appreciate/your offering/me/ride.
10. They/lucky/get/ the only two seats left/ the play.
VI/ Finish each of the following sentences in such a way that it means the same as the one printed before
it.
1. We spent five hours getting to LonDon.
It took …………
2. She doesn’t usually stay up so late.
She’s not used………………..
3. If i were you, I’d look for another job.
I suggest………………..
4. He lost his money simply because he wasn’t careful.
If…………………………….
5 They last visited me five years ago.
They haven’t ……..
6. All him suits were made in paris.
He…………..
COPOSITION
Describe your fist day at school as a child
KEYS
TEST 1
I.
1.c 5.d 9.d 13.c 17.b
2.b 6.c 10.c 14.a 18. b
3.d 7. b 11. a 15. c 19. d
4. a 8. a 12.b 16. b 20. c
II.
1. d 2. c 3. a 4. d 5. b
III.
1. b 3. c 5. b 7.c 9. a
2. a 4. d 6. a 8.d 10.c
IV.
1. difficulties 6. prepare 11. useless
2. Lectures 7. Foreign 12. Important
3. Notes 8. Word 13. Significant
4. Examinations 9. taling 14. importance
5. hard 10. Behind 15. content
V.
1. Please don’t forget to put your names on them.
2. Although they are brothers, they don’t look alike.
3. I really don’t want any dessert.
4. It’s never too late to go back to school.
5. Jeff bought a twenty – dollar sweater for half price.
6. Studying all night is good for neither your grades nor your heatth.
7. We’ll have to leave at eight o;clock to get to the office before nine.
8. Mary has losta lot of weigh on her diet.
9. I appereciate your offering me a ride.
10. They were lucky to get the only two seats left for the play.
VI.
1.It took (us) 5 hours to get to London.
2. She’s not used to staying up late.
3. I suggeest (that) you look for another job.
4. If he had been more careful, he wouldn’t have lost his money.
5. They haven’t visitted me for five years.
6. he had all his suits made in Paris.
ĐOẠN VĂN
DESCRIBE YOUR FIRST DAY AT SCHOOL AS A CHILD
That morning my mother woke up earlier than usually.She urged me to get dressed and take breafast quickly
then lenderly took me to school. On the way to school I met other children of my age accompanied by their
parents to school.
The nearer I came to the school gate, the more nervous I became. Suddenly a strange felling of fear and worry
seized my heart when I saw the majestic elementary school in front of me and I feet like crying when my
morther said good bye to me and put some coins into my pocket.
Never before had I been forced to separate from my beloved mother! But when I saw old Pupils laughing and
talking merrily I regained my composure After the droll og the drum, We were instructed to stand in line to
salute the colours and then so into our classed.A young and mild - looking teacher greeted us at the door. And
in a gentle voice, he taught us the first lesson that we never forget.
Cảm ơn bạn: Ngocthuy Le" ngocthuy17490@gmail.com đã đánh máy bài viết này
TEST 2
LEVEL: PRE- INTERMEDIATE & INTERMEDIATE
I. Choose the word or phrase which best completes each sentence.
1. He couldn't make the radio ....
a. to work c. working
b. work d. worked
2. The secretary has been busy .... all afternoon.
a. to type c. typing
b. type d. typed
3. She .... drive to the station every day but then she decided to walk instead.
a. was used to c. had used to
b. was using to d. used to
4. I'm sorry I haven't got any money. I've .... my wallet ar home.
a. left c. missed
b. forgotten d. bot a and b
5. I disapprove .... people smoking in public places.
a. with c. on
b. at d. of
6. While stady ing he was financially dependent .... his wife.
a. to c. of
b. on d. from
7. Her father won't .... drive his car.
a. allow her c. leave her
b. permit her d. let her
8. Out flight from Amsterdm to London was delayed ... the heavy fog.
a. as result c. on account for
b. because d. due to
9. Last year, Matt earned ... his brother.
a. twice as much as c. twice more than
b. twice as many as d. twice as ore as
10. He ... a bad cold, he sneezes so often.
a. has been having c. has
b. is having d. will have
11. She said she met you once at a meeting last year .... since?
a. have you met her c. did you meet her
b. had you met her d. were you met her
12. We are .... him to arriver at any moment.
a. waiting c, expecting
b, hoping d, wishing
13. I don't like to ask people for help but I wonder if you could ... me a favor.
a, make c, do
b, find d, give
14. Mr. and Mrs. Hudson are always ...with each other about money.
a, annoying c, discussing
b, arguing d, shouting
15. I would like to thank you, ... my colleagues, for the welcome you have give us.
a, on behaft of c, on account for
b, because of d, instead of
16. you ought to leave for the airport now .... there's a lot of traffic on the way.
a, in fact c, on order
b, in time d, in case
17. The children ...better leave now, it is getting late.
a, should c, would
b, had d, ought
18. he has just bought .... expensive new furniture.
a, should c, would
b, had d, ought
18. he has just bought .... expensive new furniture.
a, an c, those
b, these d, some
19. The company .... employees on strike is closing down for two weeks.
a, which c, that
b, these d, both a & c
20. It's no use .... a language if you don't try to speak it to.
a, to learn c, learning
b, learn d, learned
II. Read the passage and then choose one answer for each of the following question:
I get a lot of letter at this time of year from people complaining that they have a cold which won't go
away. thre are so many different stories about to how to prevent or cure a cold. It's often diffecult to
know what to do. Although cold is rarely dangerous, except for people who are always uncomfortabel
and usually most unpleasant, Of course you can buy lost of meducines which will help to make your
cold less unpleasant, but you must remember than nothing can actually cure a cold or make it go away
faster. Another thing is that any medicine which is strong enough to make you feel better could be
dangerous. If you are already taking drugs whether they are all right for you. And remember they
might make you sleepu - please don't try to drive if they do!Lastly, as far as avoiding cold is
concerned, whatever you may be told about magic foods or drinks, the best answer is to keep strong
and healthy - you''ll have less chance of catching a cold, and if you do, it shouldn't be so bad.
1.This is from……
a. a doctor’s notebook
b. a diary
c. a magazine
d. a school biology book
2. What is the writer’s intention?
a. To write in an amusing way.
b. To give general advice
c. To complain about his/ her health
d. To describe personal experiences.
3. Who should talk to the doctor before buying medicine for a cold?
a. people who are already talking drugs.
b. People who have never gone to the doctor’s.
c. People who have never caught a cold.
d. People who are weak.
4. What is the writer’s opinion of the “magic foods and drink”?
a. The writer doesn’t believe in “magic foods and drinks”
b. The writer strongly believes in “magic foods and drink”
c. They are of great help.
d. They are rarely dangerous.
5. Which word in the paragraph is closest in meaning to “unhealthy”?
a. good
b. strong
c. healthy
d. weak
III. Read the article below and put a cross on the letter next to the word that best fits each space.
Water is our life source. It makes up 70 percent of (1)…. bodies, and the avergage person actually
spends 18 months of his life (2)….. the bath or shower.
But we are only now learning how to look (3)……. water. Acid rain(4) ….polutted as many as
18,000 lakes and our seas and river are polluted with waste products. It is now (5)….. expensive to try
to repair the damage which has been done. We have some hope for the future, thouhg, (6)….. new
sources of water have been discovered. People (7)…. in the Sahara Desert have (8)….. fish swimming
in deep underground streams. Scientists also believe (9)….. is a hige lake underneath London. If we
have (10) ….. anything from our mistakes, we will try to keep these new areas of water clean.
1.a. the b. their c. our d. these
2. a. in b. to c. on d. at
3. a. over b. on c. after d. to
4. a. will b. has c. would d. is
5. a. very b. such c. more d. much
6. a. which b. because c. so d. even
7. a. live b. living c. to live d. lived
8. a. realized b. put c. looked d. found
9. a. it b. here c. that d. there
10. a. taught b. practiced c. known d. learnt
IV. Fill in each gap in the passage below with one suitable word.
My mother and my father are very (1)… people. Mum is always very calm not exactly easy-going,
because she doesn’t get excited. When we were small she almost never (2)….. at us. When we did
something wrong, she (3)…. to us about it very firmly, but in a calm tone of voice. If we shouted and
(4)….. , she made us go and sit by (5)….. in her sewing room until we calmed down. So when the
news came, she reacted in her (6)….. way, quietly seeing what she could do to prepare for the changes
that were coming.
Dad, on the other hand, shouted, kicked a chair, and (7)….. for a long walk to try and cool off.
During the next few days he was (8)….. with us a lot of the time, which (9)…… us, as nothing was our
fault. All of us kids were (10)….. about what was going to happen, and a bit afraid, but we didn’t talk
to our parents so much. Most of all we were (11)….. about having to (12)….. all of our school friends.
V. Use the words to make sentences.
1. We/ go/ airport/ see/ off/ last sun.
2. When/ arrive/ victim/ take/ hospital/ police
3. wish/ have/ money/ buy/ dictionary/ yesterday
4. friends/ advise/ not/ go train/ next time.
5. can/ manage/ write/ report/ yourself?
6. Would/ mind/ lend/ motorbike/ until/ week?
7. ask/ whether/ find/ solution/ problem/ yet.
8. Brown/ only/ 30 years/ director/ big company.
9. father/ tired/ so/ go/ bed/ right/ dinner.
10. It/ until/ yesterday/ police/ capture/ robber.
VI. Finish each of the following sentences in such a way that it means the same as the one printed
before it.
1.This is his first visit to England.
He is……
2. I think it may rain.
It looks as…..
3. He will come because he wants to be sure of meeting you.
He will come so…..
4. Walking in the rain gives him pleasure.
He enjoys….
5. Most of a child’s life is spent in playing.
A child spends…..
6. The fox was unsuccessful in reaching the grapes.
The fox tried in…..
COMPOSITION
Test 1
Describe your first day at school as a child.
That morning my mother woke me up earlier than usual. She urged me to get dressed and take
breakfast quickly then tenderly took me to school. On the way to schook I met other children of my
age accompanied by their parents to school.
The nearer I came to the shool gate, the more nervous I became. Suddenly a strange feeling of fead
and worry seized my heart when I saw the majestic elementary school in front of me and I felt like
crying when my mother said goodgye to me and put some coins into my pocket. Never before had I
been forced to separate from my beloved mother! But when I saw old pupils laughing and talking
merrily I regained my composure.
After the droll of a drum, we were instructed to stand in line to salute the colours and then go into
our classes. A young and mild-looking teacher greeted us at the door. And in a gentle voice, he taught
us the first lesson that we never forget!
TEST 2
Your favourite hobby
My hobby is reading books. I inherited a valuable collection of books from my deceased father and
up till now my cultural treasures have been preserved carefully.
Reading books broadens my mind and improves my knowledge. They help me distinguish the bad
from the good and prevent me from doing wrong.
Reading books brings me minutes of happiness and pleasures. I have found a source of consolation
and encouragement in books when I am in despair. They’re my true companions indeed. How relieved
and comfored I feel when I can find in good books good teachers and friends!
Instead of including myself in futile entertainments, o often absorbed in reading interestiong books
which always have miracles to remove my sorrow and hardships and reveal to me strange and new
horizons.
TEST 3
Which season of the year do you like best? Why?
I was born in Northern Vietnam, where there are four clear seasons: Spring, Summer, Autumn, and
Winter. Each season has its own charms and attractions. For example, Spring is the season which
offers beautiful flowers and warm weather; Summmer is the season of nice holidays when we can
enjoy lots of outdoor acitivities suh as picnickig, camping, swimming, fishing etc…. Autumn is so
beautiful with its ideal weather while Winter can give us such delight as gathering by a fire to chat of
tell each other funny stories. As far as I am concerned, I like Autumn best because of the following
reasons:
During Autumn, which lasts more or less from late July to early October, it is sunny most of the
time. The Autumnal sun is often splendid, dying nature yellow. The weather is also ideal because it is
not too cold nor too hot. Then the coldish northwesterly wind, which blows gently, thrills everyone
with the breath of Autumn. The leaves of the trees turning yellow or red put a goreous coat of colours
on nature. Autumn is such a wonderfuk season that many poems and songs have been
Đáp án
I.
1. b 5. d 9. A 13, c 17. B
2. c 6.b 10. C 14. B 18. d
3. d 7. D 11. B 15. A 19. B
4. a 8. C 12.c 16.d 20.c
II
1.c 2.b 3.a 4.a 5.d
III.
1.c 3.c 5. A 7.b 9.d
2. a 4. B 6. B 8. D 10. D
IV
1.defferent 5. Ourselves 9. Upset
2. shouted 6. Usual 10. Worried
3. talked 7. Went 11. Sad
4. cried 8. Cross 12. Leave
V.
1. We went to the airport to see him off last Sunday.
2. When we arrived the victim had been taken to hospital by the police.
3. I wish I had enough money to buy a dictionary yesterday.
4. My friend advised me to not to go by train next time.
5. Can you manage to write the repost by yourself?
6. Would you mind lending me your motorbile until next week?
7. He asked me whether I had found a solution to the problem yet.
8. Mr. Brown, who is only 30 yearsold, is the director of a big company.
9. My father was so tired that he wemnt to bed right after dinner.
10. It was not until yesterday that the police captured the robber.
VI
1.He is visiting England for the fist time.
2. It looks as if it’s going to rain.
3. He will come so as to be sure of meeting you.
4. He enjoys walking in the rain .
5. a child spends his/ her life playing.
6. The fox tried on vain to reach the grapes.
TEST 14
Cám ơn bạn: sao tai" taisaochu.coaibietkhong@gmail.com đã cung cấp bài viết này.
I.choose the word or phrase which best completes each sentence.
1.A good clock always keeps………. Time.
a.accurate c.certain b.serious d.true
2.she has……..a lot of money in her new job.
a.found c.earned b.done d.gained
3.would you………opening the windows now.
a.concern c.mind b.worry d.want
4.we can………difficulty without too much effort.
a.get off c.get though b.get away d.get over
5………. People go to the movies now then ten years ago.
a.fewer c.lesser b.few d.less
6.we don’t know the……….of the game.
tuyen-tap-de-thi-cong-chuc-mon-tieng-anh-co-dap-an.doc
tuyen-tap-de-thi-cong-chuc-mon-tieng-anh-co-dap-an.doc
tuyen-tap-de-thi-cong-chuc-mon-tieng-anh-co-dap-an.doc
tuyen-tap-de-thi-cong-chuc-mon-tieng-anh-co-dap-an.doc

More Related Content

What's hot

Bài tập kinh tế lượng
Bài tập kinh tế lượngBài tập kinh tế lượng
Bài tập kinh tế lượng
Juz0311
 
On tap kinh te luong co ban
On tap kinh te luong co banOn tap kinh te luong co ban
On tap kinh te luong co ban
Cam Lan Nguyen
 
Cauhoionthikinhte vĩmo
Cauhoionthikinhte vĩmoCauhoionthikinhte vĩmo
Cauhoionthikinhte vĩmo
Gia Đình Ken
 
đề cương ôn thi lịch sử học thuyêt kinh tế
đề cương ôn thi lịch sử học thuyêt kinh tếđề cương ôn thi lịch sử học thuyêt kinh tế
đề cương ôn thi lịch sử học thuyêt kinh tế
Hyo Neul Shin
 
MAN310 - PHÂN TÍCH HOẠT ĐỘNG KINH DOANH (LTTT)
 MAN310 - PHÂN TÍCH HOẠT ĐỘNG KINH DOANH (LTTT) MAN310 - PHÂN TÍCH HOẠT ĐỘNG KINH DOANH (LTTT)
MAN310 - PHÂN TÍCH HOẠT ĐỘNG KINH DOANH (LTTT)
Yen Dang
 

What's hot (20)

Bi kiep ktvm phan final
Bi kiep ktvm phan finalBi kiep ktvm phan final
Bi kiep ktvm phan final
 
Chuong5 KIỂM ĐỊNH PHI THAM SỐ
Chuong5 KIỂM ĐỊNH PHI THAM SỐChuong5 KIỂM ĐỊNH PHI THAM SỐ
Chuong5 KIỂM ĐỊNH PHI THAM SỐ
 
Ôn tập lý thuyết kinh tế lượng
Ôn tập lý thuyết kinh tế lượng Ôn tập lý thuyết kinh tế lượng
Ôn tập lý thuyết kinh tế lượng
 
Bài tập kinh tế lượng
Bài tập kinh tế lượngBài tập kinh tế lượng
Bài tập kinh tế lượng
 
đề Cương kinh tế lượng
đề Cương kinh tế lượng đề Cương kinh tế lượng
đề Cương kinh tế lượng
 
Bai tap nguyen ly thong ke
Bai tap nguyen ly thong keBai tap nguyen ly thong ke
Bai tap nguyen ly thong ke
 
Bài giảng"Kế toán nguồn vốn chủ sở hữu"
Bài giảng"Kế toán nguồn vốn chủ sở hữu"Bài giảng"Kế toán nguồn vốn chủ sở hữu"
Bài giảng"Kế toán nguồn vốn chủ sở hữu"
 
Bài tập kế toán chi phí và lời giải
Bài tập kế toán chi phí và lời giảiBài tập kế toán chi phí và lời giải
Bài tập kế toán chi phí và lời giải
 
Lựa chọn trong điều kiện không chắc chắn
Lựa chọn trong điều kiện không chắc chắnLựa chọn trong điều kiện không chắc chắn
Lựa chọn trong điều kiện không chắc chắn
 
On tap kinh te luong co ban
On tap kinh te luong co banOn tap kinh te luong co ban
On tap kinh te luong co ban
 
Cauhoionthikinhte vĩmo
Cauhoionthikinhte vĩmoCauhoionthikinhte vĩmo
Cauhoionthikinhte vĩmo
 
Bài tập kế toán tài chính 2: đầu tư tài chính
Bài tập kế toán tài chính 2: đầu tư tài chínhBài tập kế toán tài chính 2: đầu tư tài chính
Bài tập kế toán tài chính 2: đầu tư tài chính
 
Bài tập kinh tế lượng dùng eviews
Bài tập kinh tế lượng dùng eviewsBài tập kinh tế lượng dùng eviews
Bài tập kinh tế lượng dùng eviews
 
Bài tập kế toán tài chính
Bài tập kế toán tài chínhBài tập kế toán tài chính
Bài tập kế toán tài chính
 
đề cương ôn thi lịch sử học thuyêt kinh tế
đề cương ôn thi lịch sử học thuyêt kinh tếđề cương ôn thi lịch sử học thuyêt kinh tế
đề cương ôn thi lịch sử học thuyêt kinh tế
 
Trac nghiêm-va-bai-tap-nlkt-2020
Trac nghiêm-va-bai-tap-nlkt-2020Trac nghiêm-va-bai-tap-nlkt-2020
Trac nghiêm-va-bai-tap-nlkt-2020
 
MAN310 - PHÂN TÍCH HOẠT ĐỘNG KINH DOANH (LTTT)
 MAN310 - PHÂN TÍCH HOẠT ĐỘNG KINH DOANH (LTTT) MAN310 - PHÂN TÍCH HOẠT ĐỘNG KINH DOANH (LTTT)
MAN310 - PHÂN TÍCH HOẠT ĐỘNG KINH DOANH (LTTT)
 
Chương 3
Chương 3Chương 3
Chương 3
 
M.friedman
M.friedmanM.friedman
M.friedman
 
Chăm sóc khách hàng
Chăm sóc khách hàngChăm sóc khách hàng
Chăm sóc khách hàng
 

Similar to tuyen-tap-de-thi-cong-chuc-mon-tieng-anh-co-dap-an.doc

Bài tập Ngữ pháp lý thuyết (tiếng anh) có đáp án.pdf
Bài tập Ngữ pháp lý thuyết (tiếng anh) có đáp án.pdfBài tập Ngữ pháp lý thuyết (tiếng anh) có đáp án.pdf
Bài tập Ngữ pháp lý thuyết (tiếng anh) có đáp án.pdf
Man_Ebook
 
امتحان اللغة الإنجليزية 2014 نظام حديث الدور الأول , والاجابة علية
امتحان اللغة الإنجليزية 2014 نظام حديث الدور الأول , والاجابة عليةامتحان اللغة الإنجليزية 2014 نظام حديث الدور الأول , والاجابة علية
امتحان اللغة الإنجليزية 2014 نظام حديث الدور الأول , والاجابة علية
Ahmed Saleh
 
The passive answers
The passive answersThe passive answers
The passive answers
crisholm
 
creative English activity 1
creative English activity 1creative English activity 1
creative English activity 1
Jorn Buysse
 
Diagonastic test XI Year Praparatory classes 2019 PPT.pptx
Diagonastic test XI Year Praparatory classes 2019 PPT.pptxDiagonastic test XI Year Praparatory classes 2019 PPT.pptx
Diagonastic test XI Year Praparatory classes 2019 PPT.pptx
Muhammad Wajid Hussain
 
Tổng hợp đề hsg 9 + key
Tổng hợp đề hsg 9 + keyTổng hợp đề hsg 9 + key
Tổng hợp đề hsg 9 + key
Tommy Bảo
 
Soal Bahasa Inggris II Semester Ganjil (2012 2013)
Soal Bahasa Inggris II Semester Ganjil (2012 2013)Soal Bahasa Inggris II Semester Ganjil (2012 2013)
Soal Bahasa Inggris II Semester Ganjil (2012 2013)
Aji Subekti
 
الصف الاول الثانوي 2
الصف الاول الثانوي 2الصف الاول الثانوي 2
الصف الاول الثانوي 2
Ahmed Abdullah Loay
 

Similar to tuyen-tap-de-thi-cong-chuc-mon-tieng-anh-co-dap-an.doc (20)

Bài tập Ngữ pháp lý thuyết (tiếng anh) có đáp án.pdf
Bài tập Ngữ pháp lý thuyết (tiếng anh) có đáp án.pdfBài tập Ngữ pháp lý thuyết (tiếng anh) có đáp án.pdf
Bài tập Ngữ pháp lý thuyết (tiếng anh) có đáp án.pdf
 
امتحان اللغة الإنجليزية 2014 نظام حديث الدور الأول , والاجابة علية
امتحان اللغة الإنجليزية 2014 نظام حديث الدور الأول , والاجابة عليةامتحان اللغة الإنجليزية 2014 نظام حديث الدور الأول , والاجابة علية
امتحان اللغة الإنجليزية 2014 نظام حديث الدور الأول , والاجابة علية
 
The passive answers
The passive answersThe passive answers
The passive answers
 
creative English activity 1
creative English activity 1creative English activity 1
creative English activity 1
 
Taller preparatorio-icfes-inglc3a9s-convertido
Taller preparatorio-icfes-inglc3a9s-convertidoTaller preparatorio-icfes-inglc3a9s-convertido
Taller preparatorio-icfes-inglc3a9s-convertido
 
Aku
AkuAku
Aku
 
Talleres
TalleresTalleres
Talleres
 
Unit 7 practice-1
Unit 7 practice-1Unit 7 practice-1
Unit 7 practice-1
 
1- 2 unit 8 pp
1- 2 unit 8 pp1- 2 unit 8 pp
1- 2 unit 8 pp
 
1 - 2 unit 8 pp
1 - 2 unit 8 pp1 - 2 unit 8 pp
1 - 2 unit 8 pp
 
Advanced review exercises 2
Advanced review exercises 2Advanced review exercises 2
Advanced review exercises 2
 
Diagonastic test XI Year Praparatory classes 2019 PPT.pptx
Diagonastic test XI Year Praparatory classes 2019 PPT.pptxDiagonastic test XI Year Praparatory classes 2019 PPT.pptx
Diagonastic test XI Year Praparatory classes 2019 PPT.pptx
 
1st year sec paragraphs
1st year sec paragraphs  1st year sec paragraphs
1st year sec paragraphs
 
Reported speech explanation
Reported speech explanationReported speech explanation
Reported speech explanation
 
Tổng hợp đề hsg 9 + key
Tổng hợp đề hsg 9 + keyTổng hợp đề hsg 9 + key
Tổng hợp đề hsg 9 + key
 
A6 reading
A6 readingA6 reading
A6 reading
 
Soal Bahasa Inggris II Semester Ganjil (2012 2013)
Soal Bahasa Inggris II Semester Ganjil (2012 2013)Soal Bahasa Inggris II Semester Ganjil (2012 2013)
Soal Bahasa Inggris II Semester Ganjil (2012 2013)
 
2015
20152015
2015
 
Grammar Test
Grammar Test Grammar Test
Grammar Test
 
الصف الاول الثانوي 2
الصف الاول الثانوي 2الصف الاول الثانوي 2
الصف الاول الثانوي 2
 

Recently uploaded

The basics of sentences session 3pptx.pptx
The basics of sentences session 3pptx.pptxThe basics of sentences session 3pptx.pptx
The basics of sentences session 3pptx.pptx
heathfieldcps1
 
Call Girls in Uttam Nagar (delhi) call me [🔝9953056974🔝] escort service 24X7
Call Girls in  Uttam Nagar (delhi) call me [🔝9953056974🔝] escort service 24X7Call Girls in  Uttam Nagar (delhi) call me [🔝9953056974🔝] escort service 24X7
Call Girls in Uttam Nagar (delhi) call me [🔝9953056974🔝] escort service 24X7
9953056974 Low Rate Call Girls In Saket, Delhi NCR
 

Recently uploaded (20)

Basic Civil Engineering first year Notes- Chapter 4 Building.pptx
Basic Civil Engineering first year Notes- Chapter 4 Building.pptxBasic Civil Engineering first year Notes- Chapter 4 Building.pptx
Basic Civil Engineering first year Notes- Chapter 4 Building.pptx
 
COMMUNICATING NEGATIVE NEWS - APPROACHES .pptx
COMMUNICATING NEGATIVE NEWS - APPROACHES .pptxCOMMUNICATING NEGATIVE NEWS - APPROACHES .pptx
COMMUNICATING NEGATIVE NEWS - APPROACHES .pptx
 
Jamworks pilot and AI at Jisc (20/03/2024)
Jamworks pilot and AI at Jisc (20/03/2024)Jamworks pilot and AI at Jisc (20/03/2024)
Jamworks pilot and AI at Jisc (20/03/2024)
 
Mehran University Newsletter Vol-X, Issue-I, 2024
Mehran University Newsletter Vol-X, Issue-I, 2024Mehran University Newsletter Vol-X, Issue-I, 2024
Mehran University Newsletter Vol-X, Issue-I, 2024
 
21st_Century_Skills_Framework_Final_Presentation_2.pptx
21st_Century_Skills_Framework_Final_Presentation_2.pptx21st_Century_Skills_Framework_Final_Presentation_2.pptx
21st_Century_Skills_Framework_Final_Presentation_2.pptx
 
The basics of sentences session 3pptx.pptx
The basics of sentences session 3pptx.pptxThe basics of sentences session 3pptx.pptx
The basics of sentences session 3pptx.pptx
 
Food safety_Challenges food safety laboratories_.pdf
Food safety_Challenges food safety laboratories_.pdfFood safety_Challenges food safety laboratories_.pdf
Food safety_Challenges food safety laboratories_.pdf
 
Call Girls in Uttam Nagar (delhi) call me [🔝9953056974🔝] escort service 24X7
Call Girls in  Uttam Nagar (delhi) call me [🔝9953056974🔝] escort service 24X7Call Girls in  Uttam Nagar (delhi) call me [🔝9953056974🔝] escort service 24X7
Call Girls in Uttam Nagar (delhi) call me [🔝9953056974🔝] escort service 24X7
 
OSCM Unit 2_Operations Processes & Systems
OSCM Unit 2_Operations Processes & SystemsOSCM Unit 2_Operations Processes & Systems
OSCM Unit 2_Operations Processes & Systems
 
Graduate Outcomes Presentation Slides - English
Graduate Outcomes Presentation Slides - EnglishGraduate Outcomes Presentation Slides - English
Graduate Outcomes Presentation Slides - English
 
SOC 101 Demonstration of Learning Presentation
SOC 101 Demonstration of Learning PresentationSOC 101 Demonstration of Learning Presentation
SOC 101 Demonstration of Learning Presentation
 
Tatlong Kwento ni Lola basyang-1.pdf arts
Tatlong Kwento ni Lola basyang-1.pdf artsTatlong Kwento ni Lola basyang-1.pdf arts
Tatlong Kwento ni Lola basyang-1.pdf arts
 
Interdisciplinary_Insights_Data_Collection_Methods.pptx
Interdisciplinary_Insights_Data_Collection_Methods.pptxInterdisciplinary_Insights_Data_Collection_Methods.pptx
Interdisciplinary_Insights_Data_Collection_Methods.pptx
 
Towards a code of practice for AI in AT.pptx
Towards a code of practice for AI in AT.pptxTowards a code of practice for AI in AT.pptx
Towards a code of practice for AI in AT.pptx
 
Philosophy of china and it's charactistics
Philosophy of china and it's charactisticsPhilosophy of china and it's charactistics
Philosophy of china and it's charactistics
 
HMCS Vancouver Pre-Deployment Brief - May 2024 (Web Version).pptx
HMCS Vancouver Pre-Deployment Brief - May 2024 (Web Version).pptxHMCS Vancouver Pre-Deployment Brief - May 2024 (Web Version).pptx
HMCS Vancouver Pre-Deployment Brief - May 2024 (Web Version).pptx
 
FICTIONAL SALESMAN/SALESMAN SNSW 2024.pdf
FICTIONAL SALESMAN/SALESMAN SNSW 2024.pdfFICTIONAL SALESMAN/SALESMAN SNSW 2024.pdf
FICTIONAL SALESMAN/SALESMAN SNSW 2024.pdf
 
Understanding Accommodations and Modifications
Understanding  Accommodations and ModificationsUnderstanding  Accommodations and Modifications
Understanding Accommodations and Modifications
 
HMCS Max Bernays Pre-Deployment Brief (May 2024).pptx
HMCS Max Bernays Pre-Deployment Brief (May 2024).pptxHMCS Max Bernays Pre-Deployment Brief (May 2024).pptx
HMCS Max Bernays Pre-Deployment Brief (May 2024).pptx
 
Simple, Complex, and Compound Sentences Exercises.pdf
Simple, Complex, and Compound Sentences Exercises.pdfSimple, Complex, and Compound Sentences Exercises.pdf
Simple, Complex, and Compound Sentences Exercises.pdf
 

tuyen-tap-de-thi-cong-chuc-mon-tieng-anh-co-dap-an.doc

  • 1. TEST 4 Cám ơn bạn: "Nam Huy" <bui.huy.nam.1990@gmail.com> đã đánh máy bài test này. LEVEL: PRE-INTERMEDIATE & INTERMEDIATE I. Choose the word or phrase which best completes each sentence. 1. My mother can’t ____ seeing me at home all day. a. stop c. give up b. stand d. suffer 2. When the company had to lay off a number of workers, he became _____ a. considerate c. delyberate b. redundant d. dismissed 3. Dr. Johnson is a very _______ man in our neighborhood. a. impossible c. unpopular b. unlike d. disliked 4. This time next week they ________ to the United States. a. will be flying c. are flying b. will fly d. have flown 5. My uncle Tom,_______ you met the other day, is one of the best lawyers in the city. a. whose c. whom b. that d. both b and c are correct 6. Mike always keeps his promise, so you can rely ___ him. a. to c. on b. for d. at 7. They asked me ______ I had read “Moby Dick” by Herman Melville. a. what c. whether b. if d. both a and c are correct 8. I will have your bicycle ______ before returning it to you. a. repaired c. repairing b. to repair d. repair 9. Only when you are old enough ________ the truth. a. do you know c. will you know b. you do know d. you know 10. We are ______ listening to her advice. a. interested with c. tired in b. bored in d. tired of 11. He has been a war ______ for many years and covered the war in Rwanda last year. a. correspondent c. editor b. newsman d. reporter 12. She was ________ in the countryside and sent to a little village school. a. brought on c. taught b. brought up d. learned 13. The meeting was attended by nearly one hundred scientists, several of ____ were very young. a. them c. who b. those d. whom 14. He doesn’t write to his parents as often as he ______ a. used to c. got used to b. was to d. had to 15. My hair needs _____ , but I’ve been too busy to have it cut. a. being cut c. cutting b. to cut d. cut 16. Had they arrived at the fair early, they ________ what they wanted. a. would have found c. had found b. found d. find 17. After having been held ____ for 5 years, he was released yesterday. a. passive c. pensive b. active d. captive 18. You cannot ______ the truth when that detective questions you yesterday.
  • 2. a. release c. contain b. reveal d. captive 19. There is great ____ in climbing Mr.Everest, but many people have made the attempt. a. trial c. drill b. pause d. peril 20. His parents are trying to ______ him to get married. a. cause c. deserve b. determine d. persuade II. Read the passage and then choose one answer for each of the following questions: Chess must be one of the oldest games in the world. An Arab traveller in India in the year 900 wrote that it was played “long, long ago”. Chess was probably invented in India, and it has been played everywhere from Japan to Europe since 1400. The name “chess” is interesting. When one player is attacking the other’s King, he says in English, “Check”; when the King has been caught and cannot move anywhere he says “Check mate”. These words come from Persian. “Shah mat” means” the king is dead”, that is when the game is over and one player has won. Such an old game changes very slowly. The ruler have not always been the same as they are now. For example, at one time the queen could only move one square at a time. Now she is the strongest piece on the board. It would be interesting to know why this has happened! Chess takes time and thought, but it is a game for all kinds of people. You don’t have to be a champion in order to enjoy it. It is not always played by two people sitting at the same table. The first time the Americans beat the Russians was in a match played by radio. Some of the chess masters are able to play many people at the same time. The record was when one man played 400 games! It is said that some people play chess by post. This must make chess the slowest game in the world. 1. Which of the following is known to be true? a. Chess is an old Indian travelling game b. Chess is the oldest game in the world c. Chess was played in Japan and Europe before 1400 d. Chess was played in India long before 900 2. One player has won the game when the other player’s King can not move anywhere. a. he attacks the other player’s King b. he says some Persian words c. the other player’s King can not move anywhere d. he says “check” to the r other player 3. According to the old rules of the game the queen could move no more than one square at a time. a. the queen was the attacked all the time b. the King had to attacked all the time c. the queen could move no more than one square at a time d. the king could not move anywhere 4. Which of the following will you hear when one player has won the game? a. “Shah mat” c. “the King is dead” b. “check” d. “check mate” 5. Which of the following is NOT correct? a. All kinds of people can play chess. b. only two people can play chess sitting at the same table. c. Some people write each other playing chess. d. The Russians lost the game player by radio. III. Read the article below and put a cross on the letter next to the word that best fits each space. Should smoking be banned in public laces? Statistics (1) show beyond doubt that cigarette smoking can (2) damage the health, yet a surprisingly large number of people continue to smoke (3) in spite of all warnings. By doing so they are not just shortening their own lives, they are also affecting the health of (4) those around them. It is time that non-smokers fought bach! Personally, I think smoking should (5) definitely be banned in public places. In the first place, it is very unpleasant (6) to sit in a smoke-filled room, such as a restaurant or cinema, if you do not yourself smoke (7) added to this, smoking can be a serious fire risk, especially in crowed places (8) like discos. Finally, in my opinion, nobody should be asked to risk his heath just because of another person’s bad habits. Smokers may (9) protest that they should be free to do as they like. They say that we already have no-smoking areas in public places, and that this should be enough. To my mind, however, non-smokers should also be free
  • 3. to go anywhere they choose without risking their health. Smoking is harmful not just to smokers but to non-smokers too. If some people are foolish enough to continue this dangerous habits, it seems to me that they should at least be prevented (10) from doing so in public. 1. a. Survey b. Conducts c. Statistics d. Evidence 2. a. damage b. destroy c. spoil d. hurt 3. a. thanks to b. in spite of c. because of d. due to 4. a. they b. these c. those d. things 5. a. definite b. indefinite c. indefinitely d. definitely 6. a. sit b. to sit c. sitting d. sat 7. a. add b. adding c. to add d. added 8. a. for b. as c. like d. such 9. a. protest b. object c. refuse d. deserve 10. a. for b. at c. from d. in IV. Fill in each gap in the passage below with one suitable word. For over a hundred years, the (1) people of London have gone to bed and (2) got up in the morning to the deep sounds of the world (3) famous bell called Big Ben. The (4) bell appeared in Westminster Tower a year before the clock, in 1858. It (5) weighs 13.5 tons, and it is the (6) largest bell in all England. As we have (7) mentioned the clock that the bell serves appeared one year later, in 1859. Each of the clock’s four (8) faces is almost 6.5 metres (9) in deameter. The munute hand is over 4 metres (10) long and weighs over 100 kilograms. (11) Although the clock is so big, it is very exact, it is seldom more than one second (12) wrong in 1924 hours. English scientists use an original method to correct (13) mistake in the clock. There is a small tray in the middle of the pendulum.If the clock is running slow, they put a penny coin in the tray, and the clock begins to run (14) faster If the clock begins to run fast, they take (15) away the penny. The little coin is enough to make the giant clock run a second faster or slower every day. V. Use the word to make sentences. Dear Betty 1. Thank you / invite me / your birthday party. Thank you for inviting me to your birthday party. 2. I / afraid / not be able / come I’m afraid I’m not able to come 3. We / have / many things to do / this week We have got many things to do this week 4. The boss / ask / me / work overtime The boss asked me to work overtime 5. I / promise / do so / and / have to keep / promise I promise to do so and have to keep my promise 6. I / wish / can go to party I wish I coould go to the party 7. If I / be there / all of us / have good time If I were there, all of us would have good time 8. I’ll be thinking / you / when I / type / piles of letters I’ll be thinking of you when I type these piles of letters 9. I / hope / your party / a success I hope your party will be a success 10. I / wish happy returns I wish you many happy returns. VI. Finish each of the following sentences in such a way that it means the same as the one printed before it. 1. He started investigating the case a week ago. He has been investigating the case for a week. 2. I’m quite sure that she didn’t steal the necklace. She can’t have stolen the necklace. 3. The flat’s very noisy but we enjoy living there/ Even though the flat’s very noisy we enjoy living here. 4. “If I were you I wouldn’t trust Peter”, she told John.
  • 4. She advised John not to trust Peter. 5. She is a far serious student now than she used to be. She studies far more seriously now than she used to. 6. Your car does not give as much trouble as mine. Your car gives less trouble than mine. COMPOSITION Smoking damages our health (Smoking is harmful to our health) (Việc hút thuốc có hại đến sức khoẻ của chúng ta) Over the last thirty years, thanks to the efforts of various branches of science and technology, research has proved that smoking damages health and shortens man’s life-span. According to a diagram released by the World’s Health Organization (WHO), the number of people who die of diseases connected to smoking is increasing more and more. The more science and technology learn, the more damage caused by smoking is discovered. Therefore, it seems that we can never put a final stop to the list of harm caused by smoking. First of all, smoking brings about the decline of our memory because each cigarette we smoke destroys 15,000 cerebral cells. Smoking is considered the number one cause of cardio-vascular trouble, pneumonia, lung cancer etc … The main offending agents are carbon monoxide and nicotine in the smoke. These cancer – causing agents are also the ones which cause pharyngeal cancer. The toxic effects of smoke make the smoker’s voice hoarse. Additionally, the visible effect is that the tobacco or opium tar sticks to the teeth, and causes them to become dark or dirty-yellow, diminishing our good looks and charm. In conclusion, I would like to quote our well-known scholar “Le Quy Don” as saying some hundred years ago that smoking was like taking a toxic drug, the smoke of which wasted our health and diminished our life-span; yet, few people heeded his sage advice.
  • 5. TEST 15 LEVEL: PRE-INTERMEDIATE & INTERMEDIATE Cảm ơn bạn: Anh Phuong babephuong@gmail. Com đã đánh máy bài viết này: I. Choose the word or phrase which best completes each sentence. 1. He’s just arrived to find his wife in tears. a. embarrassed c. crying b. panicking d. confused 2. She’s just bought a pair of silk tights. a. pantyhose c. trousers b. gloves d. socks 3. I am scared of living on my own in a big city. a. tired c. bored b. frightened d. fond 4. I feel sorry for those who have to do that kind of work. a. pity c. apologize b. excuse d. like 5. He was living in France when the war broke out. a. ended c. occurred suddenly b. burnt out d. spread 6. The land and the house that you own are your ………… a. property c. movable possessions b. personal belongings d. savings 7. ………… are people who leave their country to live in another one. a. Deserters c. Emigrants b. Travelers d. Explorers 8. When you buy …………... in a company, you buy a part of that company. a. staff c. food b. machines d. stock 9. People ……….. their money because they want their money to grow in value. a. save c. invest b. put away d. hide 10. A(n) …………. is a person who sees something and can tell what he or she sees. a. witness c. warden b. observer d. newsreader 11. ………….. work is work which is done the same way all the time. a. Routine c. Manual b. Mental d. Office 12. When you ………. you can draw, write, compose music, or make something new. a. make up c. discover b. create d. imagine 13. Movies, sports, and reading are forms of ………… They help us relax. a. exercise c. entertainment b. study d. research 14. When a man …………. he asks a woman to marry him. a. proposes c. engages b. suggests d. offers himself 15. To …………means to helps someone remember. a. recite c. memorize b. remind d. reconsider 16. When a man ……….. a woman at her house, he goes to her house to get her. a. drops c. drops off b. picks up d. greets 17. A………… is an object that helps you remember a place you have visited a. memory c. souvenir b. diary d. note 18. When you buy tickets for the concert ………… you buy them before the time and day of the concert.
  • 6. a. in advance c. by advance b. in before d. ahead 19. People have a ………….. for special occasions, such as a wedding, a funeral, and a graduation. a. meal c. dance b. festival d. ceremony 20. When our friends have bad fortune, we try to show…………… a. love c. embarrassment b. sympathy d. pity II. Read the passage and then choose one answer for each of the following questions: During the teenage years, many young people can be difficult to talk to. They often seem to dislike being questioned. They may seem unwilling to talk about their work in school. This is a normal development at this age, though it can be very hard for parents to understand. It is part of becoming independent of teenagers trying to be adult while they are still growing up. Young people are usually more willing to talk if they believe that questions are asked out of real interest and not because people are trying to check up on them. Parents should do their best to talk to their sons and daughters about schoolwork and future plan but should not push them to talk if they don’t want to. Parents should also watch for the danger signs: some young people in trying to be adult may experiment with sex, drugs, alcohol or smoking. Parents need to watch for any sign of unusual behavior which may be connected with these and get help if necessary. 1. This passage is taken from a a. handbook for parents b. school timetable c. teenage magazine d. book for children 2. Why do adults sometimes find teenagers difficult to talk to? a. because most teenagers are quiet. b. because teenagers don’t want to talk to other people. c. because teenagers think adults are not honest. d. because most teenagers hate adults. 3. When can you expect young people to be more talkative than usual? a. When people talk to them because they are really interested and not just checking on them. b. When adults give them a lot of money to spend. c. When adults talk to them about something other than their work in school. d. When adults talk to them about sex, alcohol, and drugs. 4. Some teenagers experiment with drinking and smoking because a. cigarettes and alcohol are available everywhere. b. cigarettes and alcohol are cheap. c. women like smoking and drinking men. d. they regard them as a mark of adulthood. 5. The word BEHAVIOR in the passage most nearly means a. feeling. b. teenagers. c. activities. d. reactions. III. Read the article below and put a cross on the letter next to the word that best fits each space. The last one? After reading an article (1)…………. “Cigarette smoking and your health” I (2)………….. a cigarette to calm my (3)…………… I smoke with concentration and pleasure as I was sure that this would by my last cigarette. For a whole week I did not smoke at all and during this time, my wife suffered terribly. I had all the usual symptoms of someone giving up smoking: a bad temper and an (4)……………….. appetite. My friends kept on offering me cigarettes and cigars. They made no effort to hide their amusement whenever I produced a packet of sweets from my pocket. After seven days of this I went to a party. Everybody (5)……………… me was smoking and I felt (6)…………. uncomfortable. When my old friend Brian (7)………………… me to accept a cigarette, it was more than I could bear. I (8)………………… one guiltily, lit it and smoked with
  • 7. satisfaction. My wife was delighted that things had returned to normal once more. Anyway, as Brian pointed out, it is the easiest thing in the world to (9)…………………. up smoking. He himself has done it (10)……………..of time! 1. a. is b. entitled c. has d. made 2. a. fired b. lit c. burned d. ignited 3. a. nerves b. system c. brain d. head 4. a. small b. tiny c. enormous d. giant 5. a. over b. under c. above d. around 6. a. extremely b. not c. no d. none 7. a. prohibited b. urged c. allowed d. permitted 8. a. take b. took c. takes d. taken 9. a. quit b. abandon c. give d. stop 10. a. many b. some c. lot d. lots IV. Fill in each gap in the passage below with one suitable word. We need peace No other word in the world is so (1)…dear….. and so (2)…near.... to the (3)…hearts…..of all the common people on earth as the world (4)…peace… (5)…War….has always been harrible peace has always been (6)…desirable… We live in the atomic (7)…age....and the modern science (8)…achievements…..must serve peaceful (9)…purposes….. The use of scientific knowledge for developing newer and more horrible (10)…means… of destruction, especially the invention of the atom and hydrogen bombs has made the problem of peace and war a matter of great urgency. It is quite clear (11)…that…. a war fought with such (12)…weapons….. would lead to vast destruction in the (13)…world…. That is why the (14)…struggle…. For peace has become an urgent necessity in our time. V. Use the words to make sentences. 1. All / us took / holiday except Tom. ………………………………………… All of us took a holiday except Tom. 2. We hold / stocks / cater / your requirements. ……………………………………………………. We hold large stocks to cater for your requirements. 3. It take / me / long time / make / mind. …………………………………………………… It takes me a long time to make up my mind. 4. English courses / will not / too hard / you study. ………………………………………………………. The English course will not be too hard for you to study 5. Will not / committee decide / proposal today? ………………………………………………………. Won’t the committee decide on the proposal today? 6. We must all / pressure / authorities / if / we / want / something / do/ ……………………………………………………………………………… We must all put pressure on the authorities if we want something to be done. 7. He write / only correctly / but / neatly. …………………………………………………….. He writes not only correctly but also neatly. 8. You said / your letter / you / like / visit us / summer. ……………………………………………………………. You said in your letter that you’d like to visit us in the summer. 9. only can / computer gather / fact / also store. …………………………………………………… Not only can the computer gather facts, it can also store them. 10. She play / only / guitar / also / violin.
  • 8. …………………………………………….. She plays not only the guitar but also the violin. VI. Finish each of the following sentences in such a way that it means the same as the one printed before it. 1. She and I have never been there before. Neither…… of us has been there before …………. 2. It was such good weather that we went swimming. The weather…… was so good that we went swimming ………….. 3. I can’t see that far. It is too…… far for me to see …… 4. I have never been to the ballet before. It is…… the first time I went to the ballet …….. 5. John doesn’t always speak the truth, I’m afraid. You can’t…… believe John because he never speaks the truth …. 6. “Why don’t you put your luggage under the seat?” He asked. He suggested…… putting my luggage under the seat …………….. COMPOSITION Your English teacher ĐÁP ÁN: I. 1.c 5.c 9.c 13. c 17.c 2.a 6.a 10.a 14. a 18.a 3. b 7. c 11. a 15. b 19. d 4. a 8. d 12. b 16. b 20. b II. 1.a 2.c 3.a 4. d 5.b III. 1.b 3.a 5. d 7. b 9. c 2.b 4.c 6. a 8. b 10. d IV. 1. dear 6. desirable 11. that 2. near 7. age 12. weapons 3. hearts 8. achievements 13. world 4. peace 9. purposes 14. struggle 5. war 10. means V. 1. All of us took a holiday except Tom. 2. We hold large stocks to cater for your requirements. 3. It takes me a long time to make up my mind. 4. The English course will not be too hard for you to study 5. Won’t the committee decide on the proposal today? 6. We must all put pressure on the authorities if we want something to be done. 7. He writes not only correctly but also neatly. 8. You said in your letter that you’d like to visit us in the summer. 9. Not only can the computer gather facts, it can also store them. 10. She plays not only the guitar but also the violin.
  • 9. VI. 1. Neither of us have been there before. 2. The weather was so good that we went swimming. 3. It’s too far for me to see. 4. It is the first time I went to the ballet. 5. You can’t believe John because he never speaks the truth. 6. He suggested putting my luggage under the seat. Your English teacher My English teacher is Miss Loan. She’s of medium height, about twenty five of age. She’s fairy pretty with her oval-shaped face, straight nose, large forehead and bright eyes. Though young and attractive in appearance she’s dignified and virtuous. She’s known to be a teacher of considerable ability. Her method of teaching differs somewhat with that of our former teacher. Instead of explaining fully every new lesson beforehand she just gives us assignments and let us work out ourselves. Thus her method of teaching corresponds to all of us and help us become active in our English learning. When teaching, she particularly pays close attention to English accent and pronunciation. She often makes us pronounce an English word several times until it’s pronounced correctly and accurately. She always encourages us in our English study and thanks to her devotion to teaching us, we have been making a rapid progress. Everyone in our class is now talking about our new English teacher who deserves our love and respect.
  • 10. TEST 16 Cảm ơn bạn: "thuyhan tran" <thuyhan90@gmail.com> đã đánh máy bài viết này I. Choose the word or phrase which best completes each sentence. 1. he was sacked having an argument with his boss. a . hit b. field c. scolded d. threatened 2. I was lucky enough to find a parking …..outside the bank a. region b. field c. space d. slot 3. The dustbins are …. every Thursday a. thrown b. thrown away c. space d. emptied 4. I promptly went to sheep again a. early b. soon c. immediately d. after a white 5. English is required for that job a. compulsory b. interesting c. extra d. optional 6. Central heaving has been put in a. received b. installed c. corrected d. altered 7. Mary want to go New York by herself, but her father would not give his ….. a. control b. example c. consent d. altered
  • 11. 8. Clara wanted to buy the coat, but it cost more than she could … a. assume b. afford c. arouse d. limit 9. The show should have stared an huor ago, I don’t know what could have caused … a. alarm b. delay c. statement d. custom 10. I sometimes take John’s coat instead of my own, because the two of them look so … a. original b. similar c. comfortable d. the same 11. George had difficulty swimming across the lake, but he finally succeeded on his fourth….. a. attempt b. process c. display d. instance 12. I know I have seen that man before,I can’t … where a. assume b. wonder c. recognize d. recall 13. I want to go the library, but I’m not walking in the right … a. distance b. circumstance c. attitude d. direction 14. I want to learn more about tha American political system, but I don’t know where to get the … a. situation b. information c. conversation d. association 15. Inthis hot weather the ice will soon… a. spoil b. melt
  • 12. c. bake d. fail 16. We had hoped that Robert would agree to help us, but he has …..to a. desired b. promised c. esfused d. intended 17. Since I have been ill, my appetite has diminished a. desire for exercise b. desire for sleep c. desire for visitors d. desire for food 18. Her husband is very competent ; he will repair the roof himsefl a. talking about b. thinking about c. arguing about d. putting off 19. The time for discussing the problem is over ; now we must act a. talking about b. thinking about c. arguing about d. putting off 20. I new my father would discipline me for my action a. reward b. reprove c. congratualate d. punish II. Read the passage and the choose one answer for each of the following questions: In many old cities in Europe, there are narrow twisting roads with many shops huddling together along the two sides, These commerrcial places are not so modern and convenient as those called shopping centers in modern cities, especially in the suburbs of the big cities in the United States. Shopping centers have developed rapidly because of the shift og the population to the suburbs , the growing use of and dependence upon the automobile and the heavy tracffic in downtown areas A shopping center is a large group of stores facing a huge central enclosed mall which may be covered , heated and air-conditioned .A shopping enter is olso surrounded by a parking area with space for thousands of cars We can buy on kinds of food and get anything we need in a chiefly sold a shopping center .Unlike a supermaket, where groceries are chiefly sold , a shopping center provides us with all services besides food .We
  • 13. canget our hair cut , eyes examined , clothes washed ,We can book our tickets for a would tour and even enroll in special classes Shopping centers are , therefore , very convenient for customers , but they lack the sense of closeness as felt in older commercial center 1. The rapid development of shopping centers in mainly due to a. the fact growing prosperity of suburban paople b. the increased use of the automobile c. the growing use of heavy cars in big cities d. the shifl of the population to downtown areas 2. A shopping center is alarge group of stores facing a huge centrall mall which is a. narrow and winding b. very crowded with automobiles c. used as a store-house for heaters and conditioners d. shaded and com fortable 3. American shopping centers are especially established in the suburbs because a. the customers want to avoid the heavy traffic in downtown areas. b. the traffic is heavier in the suburbs than in the downtown areas. c. there are few people moving from the downtown areas to the neighboring regions. d. the streets in the downtown areas are so narrow and twisting. 4. Customers cant’t find the ‘sense of closeness’ in a modern shopping center because. a. all the items in the stores are very expensive. b. the shopkeepers are not very cordial. c. it is too modern and conventional. d. they worry too much about the safety of their cars. 5. In the shopping sections of many old cities in Europe, the stores are located. a. in the auburbs. b. in residential areas. c. along poor, dirty roads. d. along small, winding streets. III. Read the article below and put a cross on the letter next to the word that best fits each space. There has been a revolution in the world of newspapers. Not many years (1)……….., newspapers were still being produced using techniques unchanged for (2)……….. hundred years. The journalists gave their stories to a typist, who prepared them for an editor, who passed them on (3)……….. the printer. The printer, who was a (4)……….. skilled man, set up the type. (5)……….. was then collected to make the papes. When the pages were complete, the printing machines could be (6)………..
  • 14. Nowadays what (7)………..? the journalists type their stories into a computer. The (8)……….. checks their spelling, plans the page, shapes the articles. When the pages are ready, another computer may control the printing. (9)………..can be no doubt about it, producing a newspapers is an entinely different (10)……….. now. 1 a. before b. after c. ago d. yet 2 a. a b. some c. an d. over 3 a. to b. by a. through d. with 4 a. hardly b. mostly c. partly d. very 5 a. they b. which c. this d. all 6 a. switched b. started c. stopped d. moved 7 a. gives b. occurs c. goes d. happens 8 a. computer b. editor c. typist d. printer 9 a. it b. there c. you d. we 10 a. skill b. work c. management d. busuness IV. Fill in each gap in the passage below with one suitable word. In questions of preserving peace and saving mankind (1)……….. the threat of (2)……….. war, let one remain (3)……….. or stand (4)……….. This concerns all and everyone. Each state is large or (5)……….., socialist or (6)……….., has an important (7)……….. to make. Every responsible political party, every public organization and every person can also make an important contribution. No task is more (8)……….., more noble or humane than that of uniting all (9)……….. to achieve this lofty (10)……….. This (11)……….. must be (12)……….. by our generation, not shifted onto the shoulders of those (13)……….. will succeed us. This is the imperative of our time. This, I would say, is the (14)……….. of historic responsibility (15)……….. our decision and actions in the time remaining until the beginning of the third millennium. Mikhail Gorbachev V.Use the words to make sentences. 1. None/ us took/ holiday besides Tom. ……………………………………… 2. Mr. Jones/ cut himself/ shave. ……………………………………… 3. Please advise us/ soon/ goods have/ dispatched. ……………………………………… 4. I see/ your advertisement/ today’s “Cambridge News” ……………………………………… 5. I neither/ smoke/ drink. ……………………………………… 6. I arrive/ Monday 4th / expected/ return/ London/ Thursday/ 7th July. ……………………………………… 7. No problem/ harder/ solve/ one.
  • 15. ……………………………………… 8. The phone/ not be/ instead / I/ write/ you/ instead. ……………………………………… 9. None/ her/ friend/ wished her/ happy birthday. ……………………………………… 10. None/ students/ have/ finish/ exam yet. ……………………………………… VI. Finish each of the following sentences in such a way that it means the same as the one printed before it. 1.Given fair warning, I could have avoided that date. If you had told me ………………….. 2. Please don’t say things like that. I wish ………………….. 3. It would have been a superb weekend if it hadn’t been for weather. But ………………….. 4. Dogs can swim better than cats can. Cats cannot ………………….. 5. She hasn’t been to the hairdresser for three months. It’s ………………….. 6. There’s a spare bed in David’s room. David’s room ………………….. ĐÁP ÁN I. 1.b 5.a 9.b 13.d 17.d 2.c 6.b 10.b 14.b 18.a 3.d 7.c 11.a 15.b 19.a 4.c 8.b 12. d 16.c 20.d II. 1.b 2.d 3.a 4.c 5.d III. 1.c 3.a 5.c 7.d 9.b 2.a 4.d 6.b 8.a 10.d IV. 1.from 6.capitalist 11. task 2. nuclear 7. contribution 12. accomplished 3. indifferent 8. urgent 13. who
  • 16. 4. aloof 9. efforts 14. burden 5. small 10. goal 15. for V. 1. None of us took a holiday besides Tom 2. Mr. Jone cut himsefl shavin 3. Please advise us as soon as the goods have been despatched 4. I’ve seen (saw) your advertisement in today’s “ Cambridge News” 5. I neither smoke nor drink 6. I arrived on Monday 4th and expected to return to London on Thursday 7th July 7. No problem is harder to solve than this one 8. The phone hasn’t been installed yet, do I’m writing to you instead 9. None of her friends wished her a happy birthday 10. None of the students have finished the exam yet VI. 1. If you had told me in advance/ in (good) time I could have avoided that date. 2. T wish you wouldn’t say things like that. 3. But for the weather, it would have been a superd weekend. 4. Cat’s cannot swim as dogs (can). 5. It’s three months since she last went to the hairdresser. 6. David’s room has a spare bed in it. Describe the career you have choden, and the reasons for your choice. Choosing a career is of the most improtance. It determines one’s whole life. As for me, after careful consideration, I want to become a construction engineer. In our country, after long years of atrocious war, many houses, villages, bridges and roads were destroyed by bombing. They all need to be repaired or rebuilt. After graduation, I’ll contribute my part to the reconstruction of my war torn country into one of the most modern and powerful countries in the world. How happy I feel when I am able to turn slum areas into blocks of hight buildings, to bring happiness and joys to my poor compatriots! How proud and delighted I am go to build high-ways, bridges and factorries for my city dwellers by the sweat of my brow!
  • 17. Cảm ơn bạn: "Thu Vuong" vuongthu90@gmail.com đã đánh máy bài viết này TEST 18 LEVEL: PRE – INTERMEDIATE & INTERMEDIATE I. Choose the word or phrase which best completes each sentence. 1. She said that she……….you some day. a. had visited c. will visit b. has visited d. would visit 2. In Prague it’s better to walk and………… the atmosphere of the pretty little streets. a. feel c. think b. tough d. move 3. Mr. Jones has…………… painting since he retired. a. taken up c. taken over b. taken of d. taken in 4. Most parents find it difficult to ………… their children nowadays. a. grow up c. develop b. foster d. bring up 5. When he was running across the road, he …………….a stone. a. fell down c. fell off b. fell over d. fell into 6. “I am absolutely broke”……………………. a. So am I c. So do too b. I do too d. I feel too 7. It was a light plane. It wasn’t……………… a. dark c. black b. heavy d. deep 8. Often a team of engineers are……………… a. work on one project c. working on one project b. on one project d. to working on one project 9. Please keep the fire………………….. It’s so cold in here. a. burns c. to burn b. burn d. burning 10. I’ve been living in this village since I………………….. a. was born c. had born b. were born d. have been born 11. Tommy admitted………………… the rock through the window. a. throwing c. to throw b. being throwing d. to be throw 12. How much dows that Swiss watch……………………? a. pay c. spend b. cost d. fix 13. ………………he has never borrowed any money from me. a. Last week c. Since
  • 18. b. Up to now d. A week ago 14. I……………..to inform you that your mather died ten minutes ago. a. sorry c. apologies b. regret d. pity 15. She was a young woman. The woman was…………………. a. a you c. a young b. young d. youth 16. I can’t remember the lesson……………………… a. Either I can c. I don’t either b.Neither I can d. Neither can I 17. I………………..Bob because I didn’t invite him to my party. a. angry c. hit b. offended d. missed 18. She wears………….. a. an alarm clock c. a clock b. an alarm d. a watch 19. Someone……………..my bicycle! a. took c. takes b. has taken d. is taking 20. we call the middle of the town the……………… a. town middle c. town capital b. town heart d. town center II. Read the passage and then choose one answer for each of the following questions: The early expansion of the sugar industry was based on cane transported from two different parts of the Orient, first from India and second from the islands of the Southwest pacific. From India, sugar cane was carried through the western regions of Asia into Arabia and later into the countries bordering on the Mediterranean. It was established in Sicily in about AD 703 and was carried to spain in about AD 755. As early as AD 1150, Spain had at least 75,000 acres of cane. During this early period sugar cane was grown largely to supply local wants. The Crusades were partly responsible for the further expansion and improvement of the sugar industry and for interesting Europeans in the use of sugar. 1. From this passage, sugar cane was first grown in a. Europe c. Arabia b. the Orient d. Sicily 2. Sugar cane was first introduced into Spain in a. AD 703 c. AD 1150 b. AD 755 d. AD 1419 3. In the twelfth century, most cane was grown a. for exploration. b. for industrial uses. c. for sales in the large market cities. d. for local use. 4. One thing that caused the sugar industry to expand in Europe was a. the favorable climate in the Orient. b. the Crusades. c. better means of transportation.
  • 19. d. for local use. 5. Cane was transported a. from the islands of the Southwest Pacific, to India, to Asia and then to Spain. b. from the island of the Southwest Pacific, to the western areas of Asia and then to the countries near the Mediterranean. c. from India to the Southwest Pacific, to Asia, into Arabia and then to the countries bordering the Mediterranean. d. from India to Asia into Arabia and then to the countries on bordering the Mediterranean. III. Read the article below and put a cross on the letter next to the word that best fits each space. It was a Monday morning in the office and, as usual everyone was busy sorting through their mail before starting work. The other staff had (1)………… that Nicky, one of the computer operators, had been (2)………….. rather miserable recently and they wondered what might have happened to make her so (3)………… . Suddenly Nicky accidentally (4)………….. over a cup of coffee. Asshe was clearing up the mess, the girl sitting next to her happened to glance at Nicky’s right hand. There was a large swelling at the (5)…………… . “What on earth have you (6) ………….. to yourself? Does it (7) ………..?” asked the girl. “Yes, it does. I think you’d better go and let the doctor (9)…………. It,” said the girl. “Let’s hope there’s no permanent (10)……………. To your hand.” 1 a. recognized b. realized c. noticed d. identified 2 a. appearing b. seeming c. pretending d. looking 3 a. sick b. bored c. uninterested d. depressed 4 a. kicked b. punched c. hit d. knocked 5 a. wrist b. eblow c. knee d. ankle 6 a. made b. done c. happened d. brought 7 a. harm b. hurt c. disturb d. upset 8 a. injured b. ruined c. spoiled d. destroyed 9 a. check b. inspect c. examine d. investigate 10 a. pain b. destruction c. damage d. suffering IV. Fill in each gap in the passage below with one suitable word. Write the words in the spaces on your answer sheet. My friend Jones is not a very practical person. (1)…………… along a main road one dark night he suddenly had a flat (2)………….. Even worse he discovered that he did not have a (3)……………….wheel in the back of his car Jones waved to passing cars and lorries, but no one of them (4)…………….. Half an hour passed and he was almost (5)…………… despair. At (6)……………… he waved to a car just like his own. To his surprise, the car actually stopped and a well – dressed young woman got out. Jones was terribly (7)………….. How could a person like this possibly help him? The lady, however, offered him her own spare wheel, but Jones had to explain that he had never (8)…………… a wheel in his (9)……………She set to work at (10) ……………… and (11)……………..the wheel in a few (12)…………… while Jones looked on in admiration. V. Use the words to make sentences.
  • 20. 1. She/ secretly/ burned/ the letter/ fireplace ……………………………………………….. 2. I’ll get him/ phone right. ………………………………………………. 3. I work/ contract/ this company. ……………………………………………… 4. John should/ gone/ post office/ this morning. ………………………………………………. 5. The beat time/ call / be/ 6 p.m/ I / be always / home / that time. ……………………………………………. 6. Though he had / finished / paper / go/ sleep. …………………………………………… 7. I am / look / meet / you / airport. ……………………………………………. 8. She / very selfish woman / somehow you can’t / liking her. …………………………………………… 9. Turn left / Woodside Road and / the second / the right. ………………………………………………. 10.If she / seen / movie / she / have told you. …………………………………………… VI. Finish each of the following sentences in such a way that it means the same as the one printed before it. 1. I don’t really want them, but I’ll have them. I might…………………………. 2. He wishes he had invited her to his party. He is sorry now…………….. 3. They go out when the sun begins to rise. They …………………………… 4. Is this the cheapest carpet you’ve got? Have you got………………… 5. Nickel and silver do not differ greatly in appearance. The difference………………….. 6. He doesn’t have enough money. He’s ……………………………. ĐÁP ÁN I. 1. d 5. b 9.d 13. b 17. b 2.a 6.a 10. a 14. b 18. d 3. a 7. b 11. a 15. b 19. b 4. d 8. c 12. b 16. d 20. d
  • 21. II. 1. b 2. b 3. d 4. b 5. d III. 1. c 3. d 5. a 7. b 9. c 2. d 4. d 6. b 8. a 10. c IV. 1. driving 5. in 9. life 2. type 6. last 10. once 3. spare 7. disappointed 11. fitted 4. stopped 8. changed 12. minutes V. 1. she secretly burned the letter in the fireplace. 2. I’ll get him to the phone right away. 3. I work on contract with this company. 4. John should have gone to the post office this morning 5. The best time to call is at 6 p.m, since I am always at home by (at) that time 6. Though he had not finished the paper, he went to sleep 7. I am looking forward to meeting you at the airport. 8. She’s a very selfish woman, but somehow you can’t help liking her. 9. Turn left into Woodside Road and take the second on the right. 10. If she had seen the movie, she would have told you. VI. 1. I might as well have them 2. He is sorry now that he didn’t invite her to his party. 3. they go out in the early morning 4. have you got a cheaper carpet than this? 5. the difference in appearance between nickel and silver is not great. 6. he’s short of money. Write about a holiday you took. Give details. My friend and I recently went on a four – day holiday to Nha Trang. It was wonderful to escape the air and noise of Ho Hi Minh city. We took the night train and we were gently lulled to sleep by the sound of the train and the pleasant night air. We arrived in Nha Trang early the next morning and checked into the Grand Hotel which is very near to the beach, restaurants and nightlife. The hotle offered all kinds of accommodation from first class to economy. The staff were friendly and extremely helpful to us. They recommended a few tours which we took and we were not disappointed with. On the beach we ate fresh seafood such as boiled crabs, grilled lobsters and sanguine oysters. Nha Trang was clean and quiet, making our holiday very peaceful, restful and unforgettable.
  • 22. TEST 28 Cảm ơn bạn: Hang Cao" caohang.vcu@gmail.com đã thực hiện bài đánh máy này: Level : pre - intermediate& intermediate 1 choose the word or pharse which best completes each Sentence 1 some diseases……… quickly from one person to another. a. move c spead b.catch d get about 2 he went to factory ………….. check the progress of work a. for the purpose c so far as b. by reason of d in order to 3 why are you looking ………….. me ? she asked . a. one c at b.over d to 4 He likes to go for a holiday …….. his wife . a. without c except b. less d unless 5 I’m looking for ward to you ………….. party . a. birthday’s c birthdays’ b. birthday d birthdays 6 we’re going on a picnic ………. tomorow . a. the day after c next day after b. the next after d after 7 He was not very pleased about …………… called an incompetent i diot. a. had been c to be b. was d be ing 8 the new factory chimney was ………. Than all the trees around it . a. longe c Taller b. Greater d deeper 9 Is the …………….bread for all the sandwiches we have to make ? a. Correct c Plenty b. enought d equal 10 The was …………….enought cake for all of us to have a very mall slice. a. Just c really b. quite d hardly 11 The police – office ask us what …………….doing. a. we had c we did b. would we d we were 12 Mr white ………….the company in the way he wanted to. a.owned c ran b. rule d commanded 13 I absolutely………..wich everything that has been said . a.accept c admit b. agree d approve 14 One ………of this job is that it is near where i live a.goodness c pleasure b. prefer d advantage 15 He has a very ………. Temper and often says things he regrets later. a.angry c wram b. quickly d fast 16 The finance Minister will be making a …………. To day a bout new raters of income tax. a.talk c statement b. notice d declaration 17 I have never …………….. any experience of living in the country . a.had c done b. wished d made
  • 23. 18 pari lies …………. The river seine . a.over c next b.one d at 19 Could you please tell me if you have any elictric typewrites ………? . a.in stock c in supply b.in order d in store 20 She was unable to warm her mother that she would be late because the telephone was ……… . a.out of work c off duty b.off work d uot of odder II Read the passage and then choose one answer for each of the following questions: An eliction year is one in wich all four numbers are evenly divisible by four (1944,1948 ect .) sice 1840, america presidents elected in years ending in zero have been destined to die in office. William H.Harrison, the man who servel the shortest term, die of pneumonia Several week after his inauguration. Abraham lincoln was on of four presidents who were assassinated.he was elected in 1860 and his untimely death came just five years later. James a Garfield, a former Union army general from Ohio, was short during his first year In office (1881) by a man to whom he wouldn’t give a job. While in his second term of office (1901), William Mckinley, another Ohioan, attended the pan American exposition at Buffalo, new york.During the reception, he was assassinated while shaking hands with some of the guests. Three years after his election in 1920 warren G, Harding died in office .Althoungh it was never proved,many believe he was poisoned . Franklin D Roosevelt had been eclected four time (1932,1936,1940 and 1944), the only man to serve so long a term.He had contracted polio in 1921 and die of the illness in 1945 . John F Kennedy,the last of the line was assassinated in 1963 only three years after his eliction. Will 1980’s candidate suffer the same fate ? 1 Which of the following was not an election year ? a,1960 b,1930 c,1888 d,1824 2 which presidedent served the shortest term in office ? a, Abraham lincoln b, William Mckinley c, warren G, Harding d,William H harrison 3 Which of following is true ? a, all presidents elected in years ending in zero have died in office . b, Only presidents from Ohio have died in office. c, Franklin D .Roosevelt complated four terms as president . d, Four American presidents have been assassinated 4 How many presidents elected in years ending in zero sice 1840 have died in office ? a,7 b,5 c,4 d,3 5 Which of the following was not assassinated ? a, John F Kennedy c, Abraham lincoln b, Franklin D Roosevelt d,Jame A, Grarfield III Read the article below and put a cross on the letter next to the word that best fist each space WATER Warter is our life source .it make up 70 per cent ,of (1)…………. Bodies,and average person actually spends 18 months of his life (2) ………………… the bath of shower. But we are only now leaning how to look (3) ………………water .Acis rain (4)..................polluted with waste products .It is now (5)……………..expensive to try to repair the damage which has been done .We have some hope for the future,though,(6)……………….new sources of water have been discovered.People (7)………….in the sahara Derert have (8) ……………….fish swimming in deep underground streams. Scientists also bilieve (9)…………………… is a huge lake underneath London.If we have (10)…………… from our mistakes,we will try keep the new areas of water clean. 1a,the b their c our d these 2a in b to c on d at
  • 24. 3a over b on c after d to 4a will b has c would d is 5a very b such c more d much 6a which b because c so d even 7a live b living c to live d lived 8a realised b put c looked d found 9a it b here c that d there 10a taught b Practiced c Known d learnt VI Fill in each gap in the passage below with one suiltable word. Concorder, the world’s fattest and the (1) ………… graceful passenger plane, will soon be 25 years old,It fist flew on 2 March 1969, from Toulouse in France . Concorder(2)……….. developep (3) …………..France and Britain.From 1956 these (4)………..countries had a dream of a supersonic passenger plane the plane (5)……………over 1,5 billion on develop.It is the most tested plane in the ( 6)………… Of aviation .It was given over 5,000 hour’s of testing. Concorder flies at twice the speed of sound .This mans that it (7)…………………only 3 hours 25 minutes to fly (8) ………………Lon don and New York,compared with 7-8 hours in other(9) ………………… Each Concorde is (10) ……………at a cost of 55 million.Twenty haven been built so far Air France and British always own the most. they each have seven planes V Use the word to make sentencens Dear peter 1 I/feel/very happy /get/letter/after/such/longtime. 2 It /be /absolutely a year /since/i hear you . 3 You like /come/stay/me/countryside? 4 you be /very welcome /you want /visit. 5 What about /come end /next months ? 6 My wife /miss /you both /often speaks/you . 7 Don’t think /chirldren/love /visit /countryside? 8 Write soon /let /know/when /you /come Fondly,alane VI Finish each of the folloing sentences is such a way that it means the same as the one printed before it . 1 we must keep our school clean and tidy Our school ……………………………. 2 He stay at home .it rained hard Because ……………………………….. 3 They were selling tickets when the match started Tickets ………………………………… 4 They like fied events .They like team games They like not only ……………………... 5 They were giving the captain some presents when we left the stadium. The captain…………………………….. 6 They are glad because they have a friend like him. They are ……….to ………………………… Composition Tell about your in door hobby and why you enjoy it Đáp án 1 c 5b 9b 13b 17a 2 d 6a 10a 14d 18b 3 c 7d 11d 15b 19a 4 a 8c 12c 16c 20d II 1.b 2.a 3c 4.a 5b III 1c 3c 5a 7b 9d 2a 4.b 6.b 8d 10d IV
  • 25. 1 most 5 takes 9 planes 2 Science 6 only 10 sold 3 flight 7 both 4 was 8 between V 1 I fell very happy to get your letter after such a long time 2 If had been absolutely a year since i heard from you . 3 Would you to like to come and stay with me in the contryside ? 4 You are very wellcome if you want to visit me . 5 What about comming at the end of next mounth ? 6 My wife misses you both very much and often speaks about you . 7 Don’t you think your chirldren would love to visit the countryside . 8 Write soon and let me know when you can come VI 1 Our school must be kept clean and tidy 2 Because it rained hard ,he stayed at home 3 Tickets were being sold when the macht started 4 They like not only filed events but also team games 5 The captain was being given some prensets when we left the stadium. 6 they are glad to have a friend like him Viết đoạn văn Tell me about your indoor hobby and why you enjoy it Experience has taught me a valuable lesson,which is that i should have an in door hobby to occupy myself in my spare – time, othe wise i might spend my lei sure – time on entertaining games which are either mindless or wasteful. After careful consideration i decided to sart stamp collection since i regard postage stamps as object of interest and value .For stamps to be accepted for use as postage, stamps undergo screening steps , and only those stamps which excelled the others in meanings ,in artistic skills, and aesthetic designs will be selected .So a stamp is really a cultural trait of the country which issues it . What could be more interesting than having in my hand such a masterpiece? Stamps teach leasons on geography and history. I, of course , will have to look up from dictionaries or other books to discover mor about the origin of stamps. This will broaden my geographycal and historical knowledge more and more Then, collecting stamps is quite a relaxing activity too .I can well imagine how a new beautifuly stamp, whether it is a domestic or overseas one,Cuold make me happy whatching it for hours on end so my hobby can help me relax All in all, stamp collecting further stimulates my creative and intellectual growth,wich makes me nevers feel guilty about wasting time TEST 26 Cảm ơn bạn: na na" nana.itp@gmail.com đã thực hiện bài đánh máy này: I. Choice the word or phrase which best completes each sentence 1. Are there any good ……….. on television tonight a. Shows c. Programmes b. Screens d. Performances 2. There’s a very interesting ………… about life in the Arctic a. news c. service b. Documentary d. return 3. That’s a ………., isn’t it? I think I saw it about a fortnight ago a. second c. copy b. repeat d. return
  • 26. 4. It’s part of a ………… on living in strangge places a. set c. series b. collection d. group 5. Do you watch a lot of ………. Firm? a. travel c. tour b. journey d. sighting 6. I prefer television…………… a. stages c. acts b. screens d. dramas 7. What about …………….. programmes? a. playing c. sports b. gaming d. match 8. I like to see “Match of the Day” on Saturdays. That’s my…………… a. popular c. preferable b. favorite d. likeable 9. Is Cliff Lion still the………..? a. Presenter c. actor b. talker d. reader 10. Cliff Lion usually discusses the matches with two or three footballers in the…….. a. stage c. studio b. theatre d. floor 11. Chas Merton has asked me to write a ……………… of the programme for the Center City Daily News. a. judgement c. review b. value d. paper 12. You’ll probably see Dave on the 4:30 bus, he………… catches it to school a. sometimes c. routinely b. traditionally d. usually 13. The doctor told me to take the pills ……….. three times a day a. invariably c. customarily b. regularly d. practically 14. Timothy is …………… late when he meets Marble a. frequently c. customarily b. routinely d.practically 15. People who …………… tell lies should be punished severely a. traditionally c. practically b. habittually d. customarily 16. Would you believe it! It………. Rains for this particular festival! a. inconsistenly c. invariably b. cuatomarily d. changelessly 17. Linda…………… scores very high marks in grammar test but quite low marks in composition writing a. traditionally c. customarily b. consistently d. routinely 18. Can you tell me what time the doctor’s ………….. open? a. office c. hospital b. surgery d. ward 19. Is it for some ………… for a headache? a. improvement c. liquid b. prevention d. medicine 20. This should soon …………….. your cough a. prevent c. disappear b. solve d. cure II. Read the pasage and then choose one answer for each of the following question: An Enlish man living near Sheffield had a most unlucky day’s fishing. He was standing on a law bridge when he had the misfortune to knock his tin of bait into the river. Learning over the side of the bridge, he tried to
  • 27. hook the tin and pull it out of the river. As he did so, his car keys fell out of his pocket and disappeared in the water. Feeling thoroughly annoyed with himself and his luck, the fisherman lent over the bridge to try to see where his keys had gone. As he đi so, so bridge collapsed and he fell into the river. This was the last straw. The fisher crawled out of the river and went back to his car. There he discovered that he had locked the doors and colud not get in. His anti-thief had immobilized his car. There was nothing left for him to do but walk sadly home. 1. How did the tin of bait get into the river? a. The passage does not say b. The fisherman knocked it in accidentally. c. It was probably blown in the wind d. It fell in when the fisherman fell in. 2. When did the fisherman fist lean over the bridge? a. when it broke b. when the tin of bait fell in c. when his key fell in d. a few minutes after he first started fishing 3. What was the last straw, according to the passage. a. having to walk home b. being unable to get into his car c. the loss of his car keys d. the collapse of the bridge 4. What is the meaning of “misfortune”? a. bad luck c. accidentally b. good luck d. a clumsy act 5. What is the meaning of “thoroughly”? a. slightly c. through b. at last d. extremely III. Read the article below and put a cross on the letter next to the word that best fits each space From the moment they have the security of their accustomed environment, travelers are at risk. Environment factors such as a arduous condition, (1)…….. climate, and high altitude may cause a danger, and so may travelers’ own behavior, free from the (2)…….. of the daily routine, and determined to have a good time with little (3)………. For the consequences. When illness or injury occur abroad, travelers are again at a disadvantage from (4) ……… to communicate with a doctor on account of language and cultural difficulties or being unable to find a doctor owing to (5)…….. of the local medical system. There may be a complete (6)……….. of skilled medical care, or of medical facilities of (7)……… acceptable to travelers from technologically developed (8)……… 1. a. worsening b. unus c. sudden d. unfavorable 2. a. restraints b. assurances c. certainties d. regulation 3. a. knowledge b. awareness c. regard d. need 4. a. inability b. difficulty c. inflexibility d. timidity 5. a. miser b. doubt c. ignorance d. disbelief 6. a. breakdown b. failure c. disruption d. absence 7. a. type b. design c. standard d. degree 8. a. countries b. places c. areas d. locations IV. Fill in each gap in the passage below with one suitable word. I do my best to take care of my (1) …………. And my family’s but it’s not always easy. My wife and I (2) ……….. the cooking and shopping and I’m a bit more careful than she is not about buying foods that I don’t have colorings or other sort of (3) ……………. In. I know you can’t (4)……….. things like that entirely, but I really try to do my best. I think that chemical probably (5) …………… more health problems that most people think they do. We don’t (6)……….. and in fact don’t have many friends who do, but my mother can’t live without her cigarettes, and of course when she is here all of the air we (7)………… has some smoke in it.
  • 28. The whole family gét some kind of (8) …………….. both the kids are very keen on swimmers, my wife runs (9)……… and I play tennis at least three times a week. Acctually I think we are (10)………….. now then when we (11) ………… younger, we began (12) …….more attention to food when the kids (13) ………… young and wanted to give them a good (14) …………. By keeping fit ourselves. I certainly get fewer (15)………. Colds and flu than I did then. V. Use the words to make sentences 1. Arrange/ kitchen wares as/ mother/ do 2. Our company/ well-know/ whole saler/ ceramics. 3. She dances/ much/ beautiful/ than predecessor 4. did not you enjoy/ see her after/ many/ year? 5. The chair/ too big/ put/ that room 6. How does/ mary/ life agree/ you? 7. You/ looking/ lovely/ ever. 8. He/ only play/ piano/ also compose/ music 9. Because/ roughnessd/ road/ car would go/ furture 10. You can take / main road/ come/ crossroads/ station. VI. Finish each of the following sentences in such a way that it means the same as the one printed before it. 1. Mary told the police about the burglary Mary reported……………… 2. I never intented to go to the wedding. I never had…………………… 3. “It certainly wasn’t me took your car” said Bob Bob reused…………………………….. 4. Simon had no expected that he would feel so weak after the opẻation. The operation left…………….. 5. Although he was not guilty they executed him In spite of…………………………. 6. He was suspended for two matches for swearing at the referee Swearing at the referee earned………………. Composition: you have just arrived to visit a foreign country. Write to your parent announcing that you have arrived safely and ask them to send something to you Đáp án test 26 I. 1c 5a 9a 13b 17b 2b 6d 10c 14a 18b 3b 7c 11c 15b 19d 4c 8b 12d 16c 20d II. 1b 2b 3d 4a 5d III. 1d 2d 3c 4a 5c 6d 7c 8a
  • 29. IV. 1. health 2.do 3.chemicals 4. avoid 5. cause 6. have 7. breathe 8 sports 9. regularly 10. healthier 11. were 12. paying 13 were 14 health 15. sickness V. 1. Arrange the kitchen wares as your mother does 2. Our company is well known as a whole saler of ceramics 3. She dances much more beautifully than her predecessor 4. Didn’t you enjoy seeing her after so many years? 5. The chair is too big to put in that room 6. How does married life agree with you? 7. You’re looking lovelier than ever 8. He not only plays the piano but also composes music 9. Because of the roughness of the road, the car would not go further 10. You take the main road until you come to the crossroads near the station Composition: Dear mother and father, This letter is to let you know that I have arrived safely in Singapore. My first impressions are that Singapore is a beautiful country and that I am going to enjoy staying here very much. Everything is so clean and beautiful and very well- organized. The airport is very large and modern and I could see that they have just completed the modernization. The people working there were friendly, courteous and very helpful. They made me feel very welcome to their city. The taxi from the airport downtown was readily available and very cheap. My hotel is new modern one with every convenience. The tourist bulletin that I found in my room suggests to me so many different things that I might like to see and do, so I am sure that I will be busy every minute. Unfortunately, I forgot the shopping list that I had made out before I came here. As you know, many of my friends asked me to buy something for them. I can not possibly remember everything, so would you please send the list to me. I miss both of you and wish that you were here with me. Don’t worry about me because this is a very secure and safe place. I will write again in a few days. Your loving son,
  • 30. Cảm ơn bạn Mr Silver" <mrsilverwolf89@gmail.com đã đánh máy bài viết này: TEST 30 Level: PRE-INTERMEDIATE & INTERMEDIATE I/Choose the word or phrase which best completes each sentence: 1.It is usually better not to ……….. things, in case they are not returned. a. lose c. borrow b. offer d. lend 2. He stood on one leg, ………… against the wall, while he took off his shoe. a. leaning c. stopping b. bargain d. supporting 3. If we go to the market might find a …………. . a. trade c. stopping
  • 31. b. staying d. shopping 4. They haven’t beaten me yet. I still have one or two ………….. up my sleeve. a. traps c. tricks b. jokes d. defenses 5. My mother was …………. of making a cake when the front door bell rang. a. at the center c. halfway through b. on her way d. in te middle 6. …………., after trying three times, he passed the examination. a. Lastly c. Last b. last of all d. At last 7. The doctor told him to keep ………. Sweets and chocolate to lose weight. A. at c. off b. back b. up 8. Nobody seems to be ………….. control of those children. a. under c. with b. in d. over 9. …………. Many times I tell him, he always forgets to pass on phone messages. a. However c. Wherever b. Whatever d. Whenever 10. This is ………….. the most difficult job I’ve ever had to do. a. fells c. holds b. touches d. by myself 11. This cloth ………… very thin. a. by chance c. by heart b. by far d. by myself 12. Will the government be able to …………. All their immense difficulties? a. overcome c. dismiss b. defeat d. overtake 13. My application for a trading license was ………….. . a. held down c. put down b. turned down d. let down 14.The opening ………….. of the play took place in an army camp. a. stage c. scene b. sight d. piece 15. I have told him never …………. here again. a. come c. came b. comes d. to come 16. As soon as the consumer protection law was passed, some manufacturers began to …………. Have it changed. A, object c. campaign b. revolt d. protect 17. The BBC tries to ………….. for all tastes with its four national programmer. a. cater c. regard b. furnish d. suit 18. The tank of petrol was ………… by a carelessly discarded cigarette end. a. ignited c. exploded b. lit up d. inflamed 19. He gave me a furious look and …………… out of the room. a. surged c. stormed b. hurled d. burst 20. Unless the Prime Minister …………… the warning, inflation will rise rapidly. a. remarks c. applies b. attends d. heeds
  • 32. II. Read the passage and then chose one answer for each of the following questions: In 1972 there arrived in Philadelphia a penniless young man, eager for work and for knowledge. As the year passed, this man, Benjamin Franklin, contributed greatly to hos city and to his country. He became a printer and a publisher, and a learned man in many subjects. Ha also helped to spread learning by establishing a public library and by founding the American Philadelphia Society, which is an important academy of great scholars to this day. Franklin initiated many improvements in the city of Philadelphia, making one of the world’s first cities to have paved and lighted streets as well as a police force and a fire fighting company. He also made many practical inventions such as the Franklin stove, which was a very efficient storms. His scientific work with electricity earned Franklin world fame. Franklin played an important role in the early history of the United States. He took part in drawing up the Declaration of Independence and the Constitution. He was the first ambassador to France, and he helped negotiate the treaty of 1783, which ended the Revolutionary War. As an active member and as president of the Abolitionist Society, Franklin devoted the last years of his life to the movement to end slavery. 1. When Franklin arrived in Philadelphia in 1723, he was a. rich c. neither rich nor poor. b. well-off d. very poor. 2. How did Franklin help to spread learning? a. He was eager for work and knowledge. b. He became a printer and a publisher. c. He contributed to his city and his country. d. He established a public library and founded the American Philadelphia Society. 3. What made Franklin famous all over the world? a. The Franklin stove. b. The lightning rod. c. The fire fighting company. d. His scientific work with electricity. 4. Which of the following statements is not true? a. Franklin alone drew up the Declaration of Independence. b. Franklin was one of those eminent Americans who drew up the Declaration of Independence. c. Franklin was the first American ambassador to France. d. Franklin helped negotiate the treaty of 1783. 5. In what was Franklin active during the last years of his life? a. The Revolutionary war. b. His scientific work. c. His practical inventions. d. The abolitionist movement. III. Read the article below and put a cross on the letter next to the world that best fits each space. An Australian Mystery
  • 33. Interest in undiscovered human-like creatures to be widespread. Every one has (1) ………….. of the Yeti and its North American “cousin” Bigfoot, but since the last century there have been (2) ………….. of the existence in Australia of another, less famous creature-the Yahoo. In 1912, a Sydney newspaper (3)………….. an account by Charles Harper of a strange, large animal he observed (4) …………… the light of his campfire “It body, legs and arms were covered with long, brownish-red hair, but what (5) ………….. me as most extraordinary was its shape, which was human in some ways (6) ………….. at the same time very different. The body was enormous (7) …………… great strength. The aims were extremely long and very muscular” Harper continued: “All this observation (8) ………….. a flew minutes while the creature stood there, as if frightened by the firelight. After a flew growls and beating his breast, he (9) ………….. the first few metres upright, then on all four limbs through on the trip, a fact at which I must admit I was rather pleased” 1. a. understood b. known c. heard d. noticed 2. a. statements b. reports c. arguments d. proofs 3. a. delivered b. typed c. declared d. printed 4. a. by b. at c. with d. under 5. a. marked b. struck c. touched d. knocked 6. a. even b. just c. still d. yet 7. a. announcing b. pointing c. indicating d. engaged 8. a. lasted b. covered c. involved d. engaged 9. a. set back b. set up c. set in d. set off 10. a. should b. must c. might d. would IV. Fill in each gap in the passage below with one suitable word. Write the words in the spaces on your answer sheet. In a village on the East coast of Scotland, people were waiting anxiously for news. Two of their fishing boat (1) ……….. been caught in the storm (2) ………… had blown up during the night. In the cottages round the harbor people stood (3) ………… their doors (4) ………… worried to talk. The rest of the fishing fleet had (5) …………. The harbor before dark and the men from these ships waited and watched with the wives and families of (6) ………….. missing men. Some had (7) …………. Thick blankets and some flasks of hot drinks, knowing that the men (8) ………… be cold and tired. As dawn began to break over (9) ………… the East, a small point of light was (10) …………. In the darkness of the water and a (11) ………….. minutes later, (12) …………. Was a shout. Before long, the two boats (13) ………….. turning in past the lighthouse, to the inside of the harbor. The men (14) …………. helped out of their boats, and although they were stiff (15) ………… cold and tiredness, they were all safe. V. Use the words to make sentences. Dear Sir 1. My wife / I / now / plan / holiday / next summer. 2. We / like / go / Italy / two weeks. 3. Holiday / begin / July 15th / we return / July 29th 4. My wife / dislike / travel / coach / we / wish / go / air. 5. We / expect / stay / hotel / east coast. 6. We / grateful / you / send / information / details / charges. 7. I / trust / you / able / help / arrangements. Yours sincerely, E. Smith
  • 34. VI. Finish each of the following sentences in such a way that it means the same as the one printed before it. 1. Then she asked “Who was the first president of the United States?” She asked the student ………………………………………………………… 2. The teacher asked “Do you understand my question?” She asked the student …………………………………………….. 3. Then she said, “I didn’t tell you to go!” She told the student ……………………………………………. 4. The students answered, “I thought you asked the next student.” The student answered …………………………………………….. 5. When John arrived home, he told his mother his decision. Arriving ……………………………………….. 6. After John had watched the garbage collectors on his street, he decided he wanted to become one, too. Watching …………………………………………. COMPOSITION Describe a day spent by the river or bi the sea. TEST 30 I. 1. d 5. d 9. a 13. b 17. a 2. a 6. d 10. b 14. c 18. a 3. b 7. c 11. a 15. d 19. c 4. c 8. b 12. a 16. c 20. d II. 1. d 2. d 3. d 4. a 5. d III. 1. c 3. c 5. c 7.a 9.c 2. b 3. b 6. c 8. d 10. c IV. 1. had 6. The 11. Flew 2. which / that 7. Brought 12. There 3. at / by 8. Would 13. Were 4. and 9. In 14. Were 5. reached 10. Seen 15. With / from V. 1. My wife and I are now planning a holiday for next summer. 2. We’d like to go to Italy for two weeks. 3. Our holiday will begin on July 15th and we’ll return on July 29th . 4. My wife dislikes travelling by coach, so we wish to go by air. 5. We expect to stay at a hotel on the east coast 6. We’d be graterful if you sent us information with details of charges. 7. I trust you be able to help in these arrangements.
  • 35. VI. 1. She asked the student who the first president of the United States was. 2. She asked the student if he understood her question. 3. She told the student that she hadn’t told him to go. 4. The student answered that he had thought the teacher had asked the next student. 5. Arriving home, John told his mother his decision. 6. Watching the garbage collectors on his street, he decided he wanted to become one, too. TEST 30 Describe a day spent by the river or bi the sea. Summer comes together with its extreme heat and nobody can resist the temptation to spend their holiday at the seaside. Last week, in older to escape from the suffocating atmosphere of a crowded city, my friends and I decided to go to Vung Tau beach. When we got to the seaside, the beach had already been crammed with holiday makers. They were either sunbathing on the beach for a short tie, we all took off our clothes and let ourselves drift along on the waves. Swimming brought us a good appetite. We enjoyed some sea foods and then took a quick nap after lunch In the afternoon we strolled along the beach, inhaling in rapture the fresh sea air and took lots of photographs. A holiday at the seaside helped us get rid of all worries and trouble and gave us health and enthusiasm to resume work again. TEST 1 Cảm ơn bạn Hà Lê" thiha1989dhtn@gmail.com đã đánh máy bài viết này LEVEL: PRE – INTERMEDIATE & INTERMEDIATE I/ Choose the word or phrase which best completes each sentence. 1. A good clock always keeps…………… time. a. Certain c. accurate b. True d. serious 2. She has……………. a lot of money in her new job. a. Gained c. found b. Earned d. done 3. Would you……………. me opening the windows now?
  • 36. a. Want c. concern b. Worry d. mind 4. We can………… the difficulty without too much effort a. Get over c. get off b. Get away d. get through 5. ………… people go the movies now than ten years ago a. Less c. Few b. Lesser d. Fewer 6. We don’t know the …………….. of the game. a. Facts c. rules b. Customs d.laws 7. We had to ues our neighbor’s telephone because ours was……….. a. Out of work c. off duty b. Out of order d. off work 8. It was ………… a boring speech that I felt asleep a. Such c. very b. So d. too 9. I think you…………… better not tell him truth. a. Will c. would b. Should d. had 10. What he says makes no ……………. to me a. Reason c.sene b. Truth d. matter 11. He said that he ……………badly sick since he returned from a broad. a. Had been c. was b. Has been d. would be 12. The speaker said …………… nothings worth listenning to. a. Complete c.quite b. Almost d. completely 13. If you had come to the party, you………………. Her
  • 37. a. Would meet c. would have met b. Had met d. met 14. The manager had his seretary…………. the report for him a. Type c. typing b. Typed d. to type 15. He………….. in this office for ten years by next Monday. a. Will be working c. will have been working b. Will work d. would working 16. No sooner ……………… the office than the phone rang. a. He had left c. he was leaving b. Had he left d. was he leaving 17. Neither the director nor his assistant …………… yet. a. Have come c. has come b. Haven’t com d. hasn’t com 18. It is necessary that you …………… here at 8 a.m. tomorrow. a. Would be c. will be being b. Be d. would have been 19. The village……………. We are going to visit is far from here a. Where c. that b. Which d. both b and c are correct 20. She wishes she …………….. you the bad news yesterday. a. Didn’t tell c. hadn’t told b. Woundn’t tell d. doesn’t tell II/ Read the passage and then choose one answer for each of the following questions: A gold rush is the rapid relocation of large numbers of people to an area where gold has been discovered. Gold rushes capture the imagination and participation of many people because of the magical lure of gold and the potential for overnight affluence. The greatest gold rush in united States history was the California Gold Rush of 1849. The rush first began in 1848 when a carpenter named James Marshall discovered gold on the property of John A. Stutter in the Sacramento Valley. Hired to build a sawmill on the banks of the Amarican River, Marshall had hardly begun work when he started finding nugget after nugget gold. News of the discovery at Stuttet’s mill spread quickly, and soon thousands of persons were laying claims in the area. These people, called “ forty – niners”, rushed in from all over the world. In just two years, the population of california increased from about 26,000 to 380,000, Consequently, Californiawas officially admitted to the union as a state, in Stemper of 1850. The free – spending style of the successful miners helped to turn communities such
  • 38. as Sacramento and San Fancisco into prosperous towns. Those who were not so lucky became farmers and rachers in the Central Valley of California. 1. what is the best title for this passage? a. Famous gold Rushes. b. The forty – nines. c. The lure of gold d. The california Gold Rush of 1849. 2. According to the passage, people join gold rushes because……………….. a. They are graranteed overnight wealth. b. Gold hold a magical power for everyone. c. They believe they have a chance of becoming affuent. d. Thay have very active imaginations. 3. According to the passage, where was gold first discovered? a. At Stutter’s mill. b. On James Marshall’s property near Sacramento. c. In the streambed of the American River. d. In the central valley of california. 4. It can be inferred from the passage that the California gold rush……. a. Provided most “ forty-nines” wih long – term mining work b. Attracted people from every country in the world. c. Spead throughout the Central Valley. d. Greatly Speeded up the development of carlifornia. 5. According to the passage, what qualifire California to be admitted to the union? a. The status of Statehood. b. The great increase in population. c. The political efforts of succassful miners. d. The prosperity of California communities. III/ Read the article bolow and put a cross on the letter next tho the word that best fits each space. Shopping in the Mappstone is a must if you are visiting the area.
  • 39. There (1) …………. Many shops and services all within walking distance (2) …………. The cetal square. The city is particularly busy during (3) ……………… summer months of june, July and August when tourists visit Mappstone from all over the (4) ………………One of the main streets of the central square – Cedar Avenue – is the most popular shopping area for not (5) ………….. visittors but residents too. Stores line both sides of the avenue and often (6) …………. Open late into the evenung. Most of the shops in this avenue are small and expensive (7) …………. In nearby streets shoppers can find almost anything at more reasonable (8) …………. Tuorists wiil find that many of these shops offer and export service and goods (9) ………….. be posted direct tho the buyer’s home country. Shops ara usually closed on Mondays but ortherwise open from 9 am in the morning (10) ……….. 8 pm in the evening. 1 a. is b. are c. have d. be 2 a. of b. in c. by d. to 3 a. some b. a c. the d. one 4 a. earth b. ground c. space d. world 5 a. same b. just c. exactly d. very 6 a. stay b. go c. make d. put 7 a. since b. so c. but d. because 8 a. can b. numbers c. values d. prices 9 a. can b. vould c. might d. should 10.a. with b. up c. until d. for IV/ Fill in each gap in the passage lelow with one suitable word. One of the greattest (1) …………for foreign students in American universities is the lecture system. The professor (2) ………….and the students take (3) ……….. These notes are later used to study for (4) …………….. .If the notes aren’t good, it will be (5) ………….. for the student to (6) ……….. for a subsequent exam. It is not easy to take good notes from a lecture in a (7) …………. Language. You try to write every (8) …………. The professor says, then you are writing at the same time the professor is lecturing, this kind of note (9) means that you will fall far (10) ……….. the lecture, and the notes wiil be confused. They may be (11) …………. For later study. What can you do to take better notes? The first skill to learn is how to write only the most (12) …………. Words, not whole sentences. (13) ………….. words are the ones that present new information. The most important words are often the focus of intonation. They are usually emphssized by pith change, and this should be your signal of (14) ………. Content words, like nouns and verds, are usually be focus of information, so your notes should be almost entirely (15) …………. Words. Remember that you need information, not sentences. V/ Use the word to make sentences. 1. Please/not forget/put/names/on them. 2. although/they/brothers/they/not look/alike. 3. I/really/not want/dessert. 4. It/never/late/ go back/school.
  • 40. 5. Jeff/bought/a twenty dollars sweater/half price. 6. Study/all night/good/for/neither/your grades/nor/health. 7. we’ll/have/leave/eight o’clock/get/ office/before nine. 8. Mary/lost/ a lot/ weight/ her diet 9. I/appreciate/your offering/me/ride. 10. They/lucky/get/ the only two seats left/ the play. VI/ Finish each of the following sentences in such a way that it means the same as the one printed before it. 1. We spent five hours getting to LonDon. It took ………… 2. She doesn’t usually stay up so late. She’s not used……………….. 3. If i were you, I’d look for another job. I suggest……………….. 4. He lost his money simply because he wasn’t careful. If……………………………. 5 They last visited me five years ago. They haven’t …….. 6. All him suits were made in paris. He………….. COPOSITION Describe your fist day at school as a child
  • 41. KEYS TEST 1 I. 1.c 5.d 9.d 13.c 17.b 2.b 6.c 10.c 14.a 18. b 3.d 7. b 11. a 15. c 19. d 4. a 8. a 12.b 16. b 20. c II. 1. d 2. c 3. a 4. d 5. b III. 1. b 3. c 5. b 7.c 9. a 2. a 4. d 6. a 8.d 10.c IV. 1. difficulties 6. prepare 11. useless 2. Lectures 7. Foreign 12. Important 3. Notes 8. Word 13. Significant 4. Examinations 9. taling 14. importance 5. hard 10. Behind 15. content V. 1. Please don’t forget to put your names on them. 2. Although they are brothers, they don’t look alike.
  • 42. 3. I really don’t want any dessert. 4. It’s never too late to go back to school. 5. Jeff bought a twenty – dollar sweater for half price. 6. Studying all night is good for neither your grades nor your heatth. 7. We’ll have to leave at eight o;clock to get to the office before nine. 8. Mary has losta lot of weigh on her diet. 9. I appereciate your offering me a ride. 10. They were lucky to get the only two seats left for the play. VI. 1.It took (us) 5 hours to get to London. 2. She’s not used to staying up late. 3. I suggeest (that) you look for another job. 4. If he had been more careful, he wouldn’t have lost his money. 5. They haven’t visitted me for five years. 6. he had all his suits made in Paris. ĐOẠN VĂN DESCRIBE YOUR FIRST DAY AT SCHOOL AS A CHILD That morning my mother woke up earlier than usually.She urged me to get dressed and take breafast quickly then lenderly took me to school. On the way to school I met other children of my age accompanied by their parents to school. The nearer I came to the school gate, the more nervous I became. Suddenly a strange felling of fear and worry seized my heart when I saw the majestic elementary school in front of me and I feet like crying when my morther said good bye to me and put some coins into my pocket. Never before had I been forced to separate from my beloved mother! But when I saw old Pupils laughing and talking merrily I regained my composure After the droll og the drum, We were instructed to stand in line to salute the colours and then so into our classed.A young and mild - looking teacher greeted us at the door. And in a gentle voice, he taught us the first lesson that we never forget.
  • 43. Cảm ơn bạn: Ngocthuy Le" ngocthuy17490@gmail.com đã đánh máy bài viết này TEST 2 LEVEL: PRE- INTERMEDIATE & INTERMEDIATE I. Choose the word or phrase which best completes each sentence. 1. He couldn't make the radio .... a. to work c. working b. work d. worked 2. The secretary has been busy .... all afternoon. a. to type c. typing b. type d. typed 3. She .... drive to the station every day but then she decided to walk instead. a. was used to c. had used to b. was using to d. used to 4. I'm sorry I haven't got any money. I've .... my wallet ar home.
  • 44. a. left c. missed b. forgotten d. bot a and b 5. I disapprove .... people smoking in public places. a. with c. on b. at d. of 6. While stady ing he was financially dependent .... his wife. a. to c. of b. on d. from 7. Her father won't .... drive his car. a. allow her c. leave her b. permit her d. let her 8. Out flight from Amsterdm to London was delayed ... the heavy fog. a. as result c. on account for b. because d. due to 9. Last year, Matt earned ... his brother. a. twice as much as c. twice more than b. twice as many as d. twice as ore as 10. He ... a bad cold, he sneezes so often. a. has been having c. has b. is having d. will have 11. She said she met you once at a meeting last year .... since? a. have you met her c. did you meet her b. had you met her d. were you met her 12. We are .... him to arriver at any moment. a. waiting c, expecting b, hoping d, wishing 13. I don't like to ask people for help but I wonder if you could ... me a favor. a, make c, do b, find d, give 14. Mr. and Mrs. Hudson are always ...with each other about money. a, annoying c, discussing b, arguing d, shouting 15. I would like to thank you, ... my colleagues, for the welcome you have give us. a, on behaft of c, on account for b, because of d, instead of 16. you ought to leave for the airport now .... there's a lot of traffic on the way. a, in fact c, on order b, in time d, in case 17. The children ...better leave now, it is getting late. a, should c, would b, had d, ought 18. he has just bought .... expensive new furniture. a, should c, would b, had d, ought 18. he has just bought .... expensive new furniture. a, an c, those b, these d, some 19. The company .... employees on strike is closing down for two weeks. a, which c, that b, these d, both a & c 20. It's no use .... a language if you don't try to speak it to. a, to learn c, learning
  • 45. b, learn d, learned II. Read the passage and then choose one answer for each of the following question: I get a lot of letter at this time of year from people complaining that they have a cold which won't go away. thre are so many different stories about to how to prevent or cure a cold. It's often diffecult to know what to do. Although cold is rarely dangerous, except for people who are always uncomfortabel and usually most unpleasant, Of course you can buy lost of meducines which will help to make your cold less unpleasant, but you must remember than nothing can actually cure a cold or make it go away faster. Another thing is that any medicine which is strong enough to make you feel better could be dangerous. If you are already taking drugs whether they are all right for you. And remember they might make you sleepu - please don't try to drive if they do!Lastly, as far as avoiding cold is concerned, whatever you may be told about magic foods or drinks, the best answer is to keep strong and healthy - you''ll have less chance of catching a cold, and if you do, it shouldn't be so bad. 1.This is from…… a. a doctor’s notebook b. a diary c. a magazine d. a school biology book 2. What is the writer’s intention? a. To write in an amusing way. b. To give general advice c. To complain about his/ her health d. To describe personal experiences. 3. Who should talk to the doctor before buying medicine for a cold? a. people who are already talking drugs. b. People who have never gone to the doctor’s. c. People who have never caught a cold. d. People who are weak. 4. What is the writer’s opinion of the “magic foods and drink”? a. The writer doesn’t believe in “magic foods and drinks” b. The writer strongly believes in “magic foods and drink” c. They are of great help. d. They are rarely dangerous. 5. Which word in the paragraph is closest in meaning to “unhealthy”? a. good b. strong c. healthy d. weak III. Read the article below and put a cross on the letter next to the word that best fits each space. Water is our life source. It makes up 70 percent of (1)…. bodies, and the avergage person actually spends 18 months of his life (2)….. the bath or shower. But we are only now learning how to look (3)……. water. Acid rain(4) ….polutted as many as 18,000 lakes and our seas and river are polluted with waste products. It is now (5)….. expensive to try to repair the damage which has been done. We have some hope for the future, thouhg, (6)….. new sources of water have been discovered. People (7)…. in the Sahara Desert have (8)….. fish swimming in deep underground streams. Scientists also believe (9)….. is a hige lake underneath London. If we have (10) ….. anything from our mistakes, we will try to keep these new areas of water clean. 1.a. the b. their c. our d. these 2. a. in b. to c. on d. at 3. a. over b. on c. after d. to 4. a. will b. has c. would d. is 5. a. very b. such c. more d. much
  • 46. 6. a. which b. because c. so d. even 7. a. live b. living c. to live d. lived 8. a. realized b. put c. looked d. found 9. a. it b. here c. that d. there 10. a. taught b. practiced c. known d. learnt IV. Fill in each gap in the passage below with one suitable word. My mother and my father are very (1)… people. Mum is always very calm not exactly easy-going, because she doesn’t get excited. When we were small she almost never (2)….. at us. When we did something wrong, she (3)…. to us about it very firmly, but in a calm tone of voice. If we shouted and (4)….. , she made us go and sit by (5)….. in her sewing room until we calmed down. So when the news came, she reacted in her (6)….. way, quietly seeing what she could do to prepare for the changes that were coming. Dad, on the other hand, shouted, kicked a chair, and (7)….. for a long walk to try and cool off. During the next few days he was (8)….. with us a lot of the time, which (9)…… us, as nothing was our fault. All of us kids were (10)….. about what was going to happen, and a bit afraid, but we didn’t talk to our parents so much. Most of all we were (11)….. about having to (12)….. all of our school friends. V. Use the words to make sentences. 1. We/ go/ airport/ see/ off/ last sun. 2. When/ arrive/ victim/ take/ hospital/ police 3. wish/ have/ money/ buy/ dictionary/ yesterday 4. friends/ advise/ not/ go train/ next time. 5. can/ manage/ write/ report/ yourself? 6. Would/ mind/ lend/ motorbike/ until/ week? 7. ask/ whether/ find/ solution/ problem/ yet. 8. Brown/ only/ 30 years/ director/ big company. 9. father/ tired/ so/ go/ bed/ right/ dinner. 10. It/ until/ yesterday/ police/ capture/ robber. VI. Finish each of the following sentences in such a way that it means the same as the one printed before it. 1.This is his first visit to England. He is…… 2. I think it may rain. It looks as….. 3. He will come because he wants to be sure of meeting you. He will come so….. 4. Walking in the rain gives him pleasure. He enjoys…. 5. Most of a child’s life is spent in playing. A child spends….. 6. The fox was unsuccessful in reaching the grapes. The fox tried in….. COMPOSITION Test 1 Describe your first day at school as a child. That morning my mother woke me up earlier than usual. She urged me to get dressed and take breakfast quickly then tenderly took me to school. On the way to schook I met other children of my age accompanied by their parents to school.
  • 47. The nearer I came to the shool gate, the more nervous I became. Suddenly a strange feeling of fead and worry seized my heart when I saw the majestic elementary school in front of me and I felt like crying when my mother said goodgye to me and put some coins into my pocket. Never before had I been forced to separate from my beloved mother! But when I saw old pupils laughing and talking merrily I regained my composure. After the droll of a drum, we were instructed to stand in line to salute the colours and then go into our classes. A young and mild-looking teacher greeted us at the door. And in a gentle voice, he taught us the first lesson that we never forget! TEST 2 Your favourite hobby My hobby is reading books. I inherited a valuable collection of books from my deceased father and up till now my cultural treasures have been preserved carefully. Reading books broadens my mind and improves my knowledge. They help me distinguish the bad from the good and prevent me from doing wrong. Reading books brings me minutes of happiness and pleasures. I have found a source of consolation and encouragement in books when I am in despair. They’re my true companions indeed. How relieved and comfored I feel when I can find in good books good teachers and friends! Instead of including myself in futile entertainments, o often absorbed in reading interestiong books which always have miracles to remove my sorrow and hardships and reveal to me strange and new horizons. TEST 3 Which season of the year do you like best? Why? I was born in Northern Vietnam, where there are four clear seasons: Spring, Summer, Autumn, and Winter. Each season has its own charms and attractions. For example, Spring is the season which offers beautiful flowers and warm weather; Summmer is the season of nice holidays when we can enjoy lots of outdoor acitivities suh as picnickig, camping, swimming, fishing etc…. Autumn is so beautiful with its ideal weather while Winter can give us such delight as gathering by a fire to chat of tell each other funny stories. As far as I am concerned, I like Autumn best because of the following reasons: During Autumn, which lasts more or less from late July to early October, it is sunny most of the time. The Autumnal sun is often splendid, dying nature yellow. The weather is also ideal because it is not too cold nor too hot. Then the coldish northwesterly wind, which blows gently, thrills everyone with the breath of Autumn. The leaves of the trees turning yellow or red put a goreous coat of colours on nature. Autumn is such a wonderfuk season that many poems and songs have been Đáp án I. 1. b 5. d 9. A 13, c 17. B 2. c 6.b 10. C 14. B 18. d 3. d 7. D 11. B 15. A 19. B 4. a 8. C 12.c 16.d 20.c II 1.c 2.b 3.a 4.a 5.d III. 1.c 3.c 5. A 7.b 9.d 2. a 4. B 6. B 8. D 10. D IV 1.defferent 5. Ourselves 9. Upset 2. shouted 6. Usual 10. Worried
  • 48. 3. talked 7. Went 11. Sad 4. cried 8. Cross 12. Leave V. 1. We went to the airport to see him off last Sunday. 2. When we arrived the victim had been taken to hospital by the police. 3. I wish I had enough money to buy a dictionary yesterday. 4. My friend advised me to not to go by train next time. 5. Can you manage to write the repost by yourself? 6. Would you mind lending me your motorbile until next week? 7. He asked me whether I had found a solution to the problem yet. 8. Mr. Brown, who is only 30 yearsold, is the director of a big company. 9. My father was so tired that he wemnt to bed right after dinner. 10. It was not until yesterday that the police captured the robber. VI 1.He is visiting England for the fist time. 2. It looks as if it’s going to rain. 3. He will come so as to be sure of meeting you. 4. He enjoys walking in the rain . 5. a child spends his/ her life playing. 6. The fox tried on vain to reach the grapes. TEST 14 Cám ơn bạn: sao tai" taisaochu.coaibietkhong@gmail.com đã cung cấp bài viết này. I.choose the word or phrase which best completes each sentence. 1.A good clock always keeps………. Time. a.accurate c.certain b.serious d.true 2.she has……..a lot of money in her new job. a.found c.earned b.done d.gained 3.would you………opening the windows now. a.concern c.mind b.worry d.want 4.we can………difficulty without too much effort. a.get off c.get though b.get away d.get over 5………. People go to the movies now then ten years ago. a.fewer c.lesser b.few d.less 6.we don’t know the……….of the game.